Крок 1 - Медицина 2016 (буклет)

1 / 200
У хворого після перелому верхньої третини плечової кістки розвинувся параліч задньої групи м’язів плеча i передпліччя. Який нерв пошкоджено? The patient developed paralysis of the posterior group of muscles of the shoulder and forearm after a fracture of the upper third of the humerus. Which nerve was damaged?

М’язово-шкірний Musculocutaneous

Ліктьовий Cubit

Променевий Radial

Пахвовий Axillary

Серединний Middle

2 / 200
Хворому з ревматоїдним артритом тривалий час вводили гідрокортизон. У нього з’явилися гіперглікемія, поліурія, глюкозурія, спрага. Ці ускладнення лікування є наслідком активації процесу: A patient with rheumatoid arthritis was administered hydrocortisone for a long time. He developed hyperglycemia, polyuria, glucosuria, and thirst. These treatment complications are the result of process activation:

Глікогенез Glycogenesis

Глюконеогенез Gluconeogenesis

Гліколіз Glycolysis

Ліполіз Lipolysis

Глікогеноліз Glycogenolysis

3 / 200
Для корекції артеріального тиску при колаптоїдному стані хворому було введено мезатон. Який механізм дії даного препарату? Mezaton was administered to the patient to correct blood pressure in a colaptoid condition. What is the mechanism of action of this drug?

Стимулює β-адренорецептори Stimulates β-adrenoceptors

Блокує α-адренорецептори Blocks α-adrenoceptors

Блокує β-адренорецептори Blocks β-adrenoceptors

Стимулює α-адренорецептори Stimulates α-adrenoceptors

Стимулює α- і β-адренорецептори Stimulates α- and β-adrenoceptors

4 / 200
На електронній мікрофотографії представлені структури у вигляді відкритих міхурців, внутрішня поверхня яких вистелена одношаровим епітелієм, який утворений респіраторними та секреторними клітинами. Які це структури? The electron micrograph shows structures in the form of open vesicles, the inner surface of which is lined with a single-layered epithelium formed by respiratory and secretory cells. What are these structures?

Бронхіоли Bronchioles

Термінальні бронхіоли Terminal bronchioles

Ацинуси Acinus

Альвеолярні ходи Alveolar passages

Альвеоли Alveoli

5 / 200
В гістопрепараті представлений паренхіматозний орган, поверхневий шар кіркової речовини якого формують клубочки, утворені ендокриноцитами. Якому органу належить дана морфологічна ознака? The histopreparation shows a parenchymal organ, the surface layer of the cortical substance of which is formed by glomeruli formed by endocrinocytes. To which organ does this morphological feature belong?

Щитовидна залоза Thyroid

Наднирник Adrenal gland

Лімфатичний вузол Lymph node

Селезінка Spleen

Яєчник Ovary

6 / 200
Після переходу до змішаного харчування у новонародженої дитини виникла диспепсія з діареєю, метеоризмом, відставанням у розвитку. Біохімічна основа даної патології полягає у недостатності: After switching to a mixed diet, the newborn child developed dyspepsia with diarrhea, flatulence, and developmental delay. The biochemical basis of this pathology is a deficiency:

Сахарази та ізомальтази Sucrases and isomaltases

Ліпази та креатинкінази Lipases and creatine kinases

Трипсину та хімотрипсину Trypsin and chymotrypsin

Целюлази Cellulases

Лактази та целобіази Lactases and cellobiases

7 / 200
Хворому перед операцією було введено дитилін (лістенон) і проведено інтубацію. Дефіцит якого ферменту в організмі хворого подовжує дію м’язового релаксанту? Before the operation, the patient was administered ditilin (Listenone) and intubated. The deficiency of which enzyme in the patient's body prolongs the effect of the muscle relaxant?

N-ацетилтрансфераза N-acetyltransferase

Карбангідраза Carbonhydrase

Сукцинатдегідрогеназа Succinate dehydrogenase

K-Na-АТФ-аза K-Na-ATPase

Псевдохолінестераза Pseudocholinesterase

8 / 200
У хворого із захворюванням печінки виявлено зниження вмісту протромбіну в крові. Це призведе, перш за все, до порушення: In a patient with liver disease, a decrease in the content of prothrombin in the blood was detected. This will lead, first of all, to a violation:

Першої фази коагуляційного гемостазу First phase of coagulation hemostasis

Фібринолізу Fibrinolysis

Другої фази коагуляційного гемостазу Second phase of coagulation hemostasis

Судинно-тромбоцитарного гемостазу Vascular-platelet hemostasis

Антикоагулянтних властивостей крові Anticoagulant properties of blood

9 / 200
У хворого із сечокам’яною хворобою виникли нестерпні спастичні болі. Для попередження больового шоку йому ввели разом з атропіном наркотичний анальгетик, що не має спазмогенного ефекту. Який це був препарат? A patient with urolithiasis developed intolerable spastic pains. To prevent painful shock, he was administered a narcotic analgesic with atropine, which does not have a spasmogenic effect. What was it preparation?

Пірітрамід Piritramide

Промедол Promedol

Етилморфіну гідрохлорид Ethylmorphine hydrochloride

Трамадол Tramadol

Морфіну гідрохлорид Morphine hydrochloride

10 / 200
Яким буде скорочення м’язів верхньої кінцівки при намаганні підняти непосильний вантаж? What will be the contraction of the muscles of the upper limb when trying to lift an overwhelming load?

Фазичне Phase

Ауксотонічне Auxotonic

Ізотонічне Isotonic

Одиночне Single

Ізометричние Isometric

11 / 200
Для лікування урогенітальних інфекцій використовують хінолони - інгібітори ферменту ДнК-гірази. Який процес порушується під дією хінолонів у першу чергу? Quinolones are used to treat urogenital infections - inhibitors of the DnK-gyrase enzyme. What process is disrupted by quinolones in the first place?

Ампліфікація генів Gene Amplification

Рекомбінація генів Gene recombination

Зворотна транскрипція Reverse transcription

Репарація ДНК DNA Repair

Реплікація ДНК DNA Replication

12 / 200
Жінку госпіталізовано в клініку з симптомами гострого живота. При обстеженні виникла підозра на позаматкову вагітність. Яке з анатомічних утворень таза необхідно пропунктувати для підтвердження діагнозу? A woman was admitted to the clinic with symptoms of an acute abdomen. An ectopic pregnancy was suspected during the examination. Which of the pelvic anatomical formations must be identified to confirm the diagnosis?

Excavatio vesicouterina Excavatio vesicouterina

Excavatio rectouterina Excavatio rectouterina

Excavatio rectovesicalis Excavatio rectovesicalis

Processus vaginalis peritonei Processus vaginalis peritonei

Fossa ischiorectalis Fossa ischiorectalis

13 / 200
Щуру в плевральну порожнину введено 0,5 мл повітря. Який тип недостатності дихання виникає в даному випадку? 0.5 ml of air was injected into the pleural cavity of a rat. What type of respiratory failure occurs in this case?

Дисрегуляторне порушення альвеолярної вентиляції Dysregulatory disorder of alveolar ventilation

Обструктивне порушення альвеолярної вентиляції Obstructive violation of alveolar ventilation

Перфузійний Perfusion

Рестриктивне порушення альвеолярної вентиляції Restrictive violation of alveolar ventilation

Дифузійний Diffuse

14 / 200
Дитина 10-ти років страждає на стафілококовий дерматит. Лікування бензилпеніциліном не дало результатів. Призначення комбінованого препарату пеніциліну з клавулановою кислотою дало швидке одужання. Яка причина позитивної дії цього препарату? A 10-year-old child suffers from staphylococcal dermatitis. Treatment with benzylpenicillin did not give results. The appointment of a combined penicillin with clavulanic acid resulted in a quick recovery. What is the reason for the positive effect of this drug ?

Активація фосфодіестерази Activation of phosphodiesterase

Гальмування аденозиндезамінази Inhibition of adenosine deaminase

Інактивація β-лактамази Inactivation of β-lactamase

Гальмування транспептидази Inhibition of transpeptidase

Блокада транслокази Translocase blockade

15 / 200
У хворого спостерігається типова для нападу малярії клінічна картина: озноб, жар, проливний піт. Яка стадія малярійного плазмодію найімовірніше буде виявлена в крові хворого в цей час? The patient has a clinical picture typical of a malaria attack: chills, fever, profuse sweat. What stage of malaria plasmodium is most likely to be detected in the patient's blood at this time?

Спорозоїт Sporozoite

Мерозоїт Merozoite

Спороциста Sporocyst

Мікро- або макрогамети Micro or macro gametes

Оокінета Ookineta

16 / 200
У немовляти спостерігаються епілептиформні судоми, викликані дефіцитом вітаміну В6. Це спричинено зменшенням у нервовій тканині гальмівного медіатора - γ-аміномасляної кислоти. Активність якого ферменту знижена при цьому? A baby has epileptiform seizures caused by a deficiency of vitamin B6. This is caused by a decrease in the nerve tissue of the inhibitory mediator - γ-aminobutyric acid. The activity of which enzyme is reduced?

Глутаматдекарбоксилаза Glutamate decarboxylase

Глутаматдегідрогеназа Glutamate dehydrogenase

Глутаматсинтетаза Glutamate synthetase

Аланінамінотрансфераза Alanine aminotransferase

Піридоксалькіназа Pyridoxal Kinase

17 / 200
Жінку 44-х років вжалила оса внаслідок чого розвинувся шок. В анамнезі вже була важка алергічна реакція на жалення оси. Об’єктивно: пульс -179/хв., слабкий, АТ- 80/40 ммрт.ст., ЧД-26/хв. Яка провідна ланка патогенезу анафілактичного шоку? A 44-year-old woman was stung by a wasp, as a result of which she developed shock. She already had a severe allergic reaction to a wasp sting in the anamnesis. Objectively: pulse -179/min. , weak, blood pressure - 80/40 mmHg, blood pressure - 26/min. What is the leading link in the pathogenesis of anaphylactic shock?

Зменшення об’єму циркулюючої крові Decreasing the volume of circulating blood

Зниження периферійного опору судин Reduction of peripheral vascular resistance

Біль Pain

Тахікардія Tachycardia

Зменшення ударного об’єму серця Decreased stroke volume of the heart

18 / 200
У препараті в одній з судин мікроциркуляторного русла середня оболонка утворена 1-2 шарами гладеньких міоцитів, які розташовані поодинці і мають спіралеподібний напрямок. Зовнішня оболонка представлена тонким шаром пухкої волокнистої сполучної тканини. Вкажіть вид судини: In the preparation, in one of the vessels of the microcirculatory bed, the middle membrane is formed by 1-2 layers of smooth myocytes, which are located alone and have a spiral direction. The outer membrane is represented by a thin layer of loose fibrous connective tissue. Specify the type of vessel:

Артеріоловенулярний анастомоз Arteriovenular anastomosis

Посткапіляр Postcapillary

Капіляр Capillary

Артеріола Arteriole

Венула Venula

19 / 200
Фекалії дитини, що хворіє на ентерит, емульгують в фізіологічному розчині і краплю емульсії наносять на елективне середовище: 10% молочно-сольовий, або жовтково-сольовий агар. Який мікроорганізм передбачається виділити? Feces of a child suffering from enteritis are emulsified in a physiological solution and a drop of the emulsion is applied to an elective medium: 10% milk-salt or yolk-salt agar. Which the microorganism is supposed to be isolated?

Кишкова паличка Escherichia coli

Стафілокок Staphylococcus

Клебсієла Klebsiella

Стрептокок Streptococcus

Ентерокок Enterococcus

20 / 200
При обстеженні хворого виявлена характерна клініка колагенозу Вкажіть, збільшення якого показника сечі характерне для цієї патології: During the examination of the patient, a characteristic collagenosis clinic was revealed. Indicate which increase in the urine indicator is characteristic of this pathology:

Солі амонію Ammonium salts

Мінеральні солі Mineral salts

Гідроксипролін Hydroxyproline

Аргінін Arginine

Глюкоза Glucose

21 / 200
При патологічних процесах, які супроводжуються гіпоксією, відбувається неповне відновлення молекули кисню в дихальному ланцюзі і накопичення пероксиду водню. Вкажіть фермент, який забезпечує його руйнування: In pathological processes that are accompanied by hypoxia, there is an incomplete restoration of the oxygen molecule in the respiratory chain and the accumulation of hydrogen peroxide. Specify the enzyme that ensures its destruction:

Аконітаза Aconitase

Сукцинатдегідрогеназа Succinate dehydrogenase

Кетоглутаратдегідрогеназа Ketoglutarate dehydrogenase

Каталаза Catalase

Цитохромоксидаза Cytochrome oxidase

22 / 200
Електрофоретичне дослідження сироватки крові хворого пневмонією показало збільшення одної з білкових фракцій. Вкажіть її: Electrophoretic examination of the blood serum of a pneumonia patient showed an increase in one of the protein fractions. Specify it:

α2 -глобуліни α2 -globulins

Альбуміни Albumins

α1 -глобуліни α1 -globulins

β-глобуліни β-globulins

γ-глобуліни γ-globulins

23 / 200
При травмі в області тазу у хворого на рентгенологічному знімку виявлено некроз головки стегнової кістки. Під час травми кульшового суглоба було пошкоджено зв’язку: With an injury in the pelvic area, necrosis of the femoral head was detected in the patient's X-ray image. During the injury of the hip joint, the ligament was damaged:

Головки стегнової кістки Femoral heads

Клубово-стегнова Iliofemoral

Сіднично-стегнова Gluteal-femoral

Лобкова-стегнова pubic-femoral

- -

24 / 200
Людині внутрішньовенно ввели 0,5 л ізотонічного розчину лікарської речовини. Які з рецепторів насамперед прореагують на зміни водно-сольового балансу організму? A person intravenously injected 0.5 l of an isotonic solution of a medicinal substance. Which of the receptors will first of all react to changes in the body's water-salt balance?

Осморецептори печінки Osmoreceptors of the liver

Осморецептори гіпоталамусу Osmoreceptors of the hypothalamus

Волюморецептори порожнистих вен і передсердь Volume receptors of the vena cava and atria

Натрієві рецептори гіпоталамуса Sodium receptors of the hypothalamus

Барорецептори дуги аорти Baroreceptors of the aortic arch

25 / 200
В судово-медичній експертизі широко використовується метод дактилоскопії, який оснований на тому, що сосочковий шар дерми визначає строго індивідуальний малюнок на поверхні шкіри. Яка тканина утворює цей шар дерми? The dactyloscopy method is widely used in forensic medical examination, which is based on the fact that the papillary layer of the dermis determines a strictly individual pattern on the surface of the skin. What tissue forms this layer of the dermis ?

Ретикулярна тканина Reticular tissue

Жирова тканина Adipose tissue

Пухка волокниста неоформлена сполучна частина Fluffy fibrous rough connector

Щільна оформлена сполучна тканина Dense designed connective tissue

Щільна неоформлена сполучна тканина Dense unformed connective tissue

26 / 200
Важливою складовою частиною ниркового фільтраційного бар’єру є тришарова базальна мембрана, яка має спеціальну сітчасту будову її середнього електроннощільного шару. Де міститься ця базальна мембрана? An important component of the kidney's filtration barrier is the three-layer basement membrane, which has a special mesh structure of its middle electron-dense layer. Where is this basement membrane located?

Дистальні прямі канальці Distal straight tubules

Проксимальні канальці Proximal tubules

Ниркове тільце Renal corpuscle

Тонкі канальці Thin tubules

Капіляри перитубулярної капілярної сітки Capillaries of the peritubular capillary network

27 / 200
Чоловік 42-х років помер при явищах вираженої інтоксикації і дихальної недостатності. На розтині: тканина легень у всіх відділах строката, з множинними дрібновогнищевими крововиливами та вогнищами емфіземи. Гістологічно у легенях: геморагічна бронхопневмонія з абсцедуванням, у цитоплазмі клітин епітелію бронхів еозинофільні і базофільні включення. Діагностуйте виявлене на секції захворювання: A 42-year-old man died due to symptoms of severe intoxication and respiratory failure. At autopsy: lung tissue in all departments is variegated, with multiple small focal hemorrhages and foci of emphysema. Histologically in the lungs: hemorrhagic bronchopneumonia with abscessation, eosinophilic and basophilic inclusions in the cytoplasm of the cells of the bronchial epithelium. Diagnose the disease detected on the section:

Грип Flu

Стафілококова бронхопневмонія Staphylococcal bronchopneumonia

Крупозна пневмонія Croup pneumonia

Часткова пневмонія Partial pneumonia

Плевропневмонія Pleuropneumonia

28 / 200
В експерименті подразнюють скелетний м’яз серією електричних імпульсів. Який вид м’язового скорочення виникне, якщо кожний наступний імпульс припадає на період вкорочення поодинокого м’язового скорочення? In an experiment, a skeletal muscle is stimulated with a series of electrical impulses. What type of muscle contraction will occur if each subsequent impulse falls within the period of shortening of a single muscle contraction?

Контрактура м’яза Muscle contracture

Зубчастий тетанус Serrated tetanus

Суцільний тетанус Solid tetanus

Серія поодиноких скорочень Series of single abbreviations

Асинхронний тетанус Asynchronous tetanus

29 / 200
У чоловіка 53-х років діагностовано сечокам’яну хворобу з утворенням уратів. Цьому пацієнту призначено аллопурінол, який є конкурентним інгібітором ферменту: A 53-year-old man was diagnosed with urolithiasis with urate formation. This patient was prescribed allopurinol, which is a competitive enzyme inhibitor:

Дигідроурацилдегідрогеназа Dihydrouracil dehydrogenase

Уреаза Urease

Уратоксидаза Urate oxidase

Уриділтрансфераза Uridyltransferase

Ксантиноксидаза Xanthine oxidase

30 / 200
Мати зауважила занадто темну сечу у її 5-річної дитини. Дитина скарг не висловлює. Жовчних пігментів у сєчі не виявлено. Поставлено діагноз алкаптонурія. Дефіцит якого ферменту має місце у дитини? The mother noticed too dark urine in her 5-year-old child. The child does not complain. Bile pigments were not detected in the urine. A diagnosis of alkaptonuria was made. What enzyme deficiency occurs does the child have?

Оксидаза гомогентизинової кислоти Homogentisic acid oxidase

Оксидаза оксифенілпірувату Oxyphenylpyruvate oxidase

Тирозиназа Tyrosinase

Фенілаланінгідроксилаза Phenylalanine hydroxylase

Декарбоксилаза фенілпірувату Phenylpyruvate decarboxylase

31 / 200
До дерматолога звернулася пацієнтка із скаргами на екзематозне ураження шкіри рук, що з’являється після контакту з миючим засобом ”Лотос” Використання гумових рукавичок запобігає цьому. Патологічна реакція шкіри зумовлена активацією: A patient turned to a dermatologist with complaints of eczematous damage to the skin of the hands, which appeared after contact with the detergent 'Lotus'. The use of rubber gloves prevents this. Pathological reaction of the skin due to activation:

Нейтрофілів Neutrophils

Т-лімфоцитів T-lymphocytes

Базофілів Basophils

В-лімфоцитів B-lymphocytes

Моноцитів Monocytes

32 / 200
У хворої 36-ти років, яка лікувалася сульфаніламідами з приводу респіраторної вірусної інфекції, в крові гіпорегенераторна нормохромна анемія, лейкопенія, тромбоцитопенія. В кістковому мозку - зменшення кількості мієлокаріоцитів. Яка це анемія? A 36-year-old patient, who was treated with sulfonamides for a respiratory viral infection, has hyporegenerative normochromic anemia, leukopenia, thrombocytopenia in the blood. In the bone marrow - a decrease in the number of myelokaryocytes . What kind of anemia is this?

Залізодефіцитна Iron deficiency

Гіпопластична Hypoplastic

Гемолітична Hemolytic

В12-фолієводефіцитна B12-folate deficient

Постгеморагічна Posthemorrhagic

33 / 200
Чоловік 38-ми років раптово помер. На розтині: у задній стінці лівого шлуночка серця виявлено інфаркт міокарда. Які найбільш імовірні зміни у будові міокардіоцитів можна побачити у вогнищі інфаркту мікроскопічно? A 38-year-old man died suddenly. At autopsy: a myocardial infarction was found in the back wall of the left ventricle of the heart. What are the most likely changes in the structure of myocardiocytes that can be seen in the heart of the infarction microscopically?

Вуглеводна дистрофія Carbohydrate dystrophy

Звапнування Calcification

Білкова дистрофія Protein dystrophy

Каріолізис Karyolysis

Жирова дистрофія Fat dystrophy

34 / 200
У відділення реанімації надійшов чоловік 47-ми років з діагнозом інфаркт міокарда. Яка з фракцій лактатдегідрогенази (ЛДГ) буде переважати в сироватці крові впродовж перших двох діб захворювання? A 47-year-old man was admitted to the intensive care unit with a diagnosis of myocardial infarction. Which fraction of lactate dehydrogenase (LDH) will predominate in the blood serum during the first two days of the disease?

ЛДГ2 LDH2

ЛДГ1 LDH1

ЛДГ4 LDH4

ЛДГ5 LDH5

ЛДГ3 LDH3

35 / 200
У хлопчика 2-х років спостерігається збільшення в розмірах печінки та селезінки, катаракта. В крові підвищена концентрація цукру, але тест толерантності до глюкози в нормі. Спадкове порушення обміну якої речовини є причиною цього стану? A 2-year-old boy has an increase in the size of the liver and spleen, a cataract. The concentration of sugar in the blood is increased, but the glucose tolerance test is normal. Hereditary metabolic disorder what substance is the cause of this condition?

Фруктоза Fructose

Сахароза Sucrose

Глюкоза Glucose

Галактоза Galactose

Мальтоза Maltose

36 / 200
У неврологічне відділення з приводу мозкового крововиливу поступив хворий 62-х років. Об’єктивно: стан важкий. Спостерігається наростання глибини і частоти дихання, а потім його зменшення до апное, після чого цикл дихальних рухів відновлюється. Який тип дихання у хворого? A 62-year-old patient was admitted to the neurology department due to cerebral hemorrhage. Objectively: the condition is severe. An increase in the depth and frequency of breathing is observed, and then it decreases to apnea, after which the cycle of respiratory movements is restored. What type of breathing does the patient have?

Гаспінг-дихання Gasping Breath

Кусмауля Kusmaul

Чейна-Стокса Cheyne-Stokes

Апнейстичне Apneistic

Біота Biota

37 / 200
Глікоген, що надійшов з їжею, гідролізується у шлунково-кишковому тракті. Який кінцевий продукт утворюється в результаті цього процесу? Dietary glycogen is hydrolyzed in the gastrointestinal tract. What is the end product of this process?

Глюкоза Glucose

Лактоза Lactose

Лактат Lactate

Фруктоза Fructose

Галактоза Galactose

38 / 200
Після перенесеного геморагічного інсульту у хворого розвинулася кіста головного мозку. Через 2 роки помер від післягрипозної пневмонії. На секції трупа виявлено в мозку кісту із стінками біло-іржавого відтінку, реакція Перлса позитивна. Який з процесів найбільш імовірний у стінці кісти? After a hemorrhagic stroke, the patient developed a brain cyst. 2 years later, he died of post-influenza pneumonia. An autopsy revealed a brain cyst with white-rusty walls, the Perls reaction is positive. Which of the processes is most likely in the cyst wall?

Загальний гемосидероз General hemosiderosis

Місцевий гемосидероз Local hemosiderosis

Місцевий гемомеланоз Local hemomelanosis

Первинний гемохроматоз Primary hemochromatosis

Інфільтрація білірубіну Bilirubin infiltration

39 / 200
При розтині тіла померлого чоловіка 73-х років, який довго страждав на ішемічну хворобу серця з серцевою недостатністю, знайдено: ’’мускатна” печінка, бура індурація легень, ціанотична індурація нирок та селезінки. Який з видів порушення кровообігу найбільш імовірний? At the autopsy of the dead man of 73, who suffered from coronary heart disease with heart failure for a long time, the following were found: 'nutmeg' liver, brown induration of the lungs, cyanotic induration of the kidneys and spleen. Which type of circulatory disturbance is most likely?

Артеріальна гіперемія Arterial hyperemia

Хронічне загальне венозне повнокрів’я Chronic general venous congestion

Гостре загальне венозне повнокрів’я Acute general venous congestion

Хронічне малокрів’я Chronic anemia

Гостре малокрів’я Acute anemia

40 / 200
У нейрохірургічне відділення поступив 54-річний чоловік із скаргами на відсутність чутливості шкіри нижньої повіки, латеральної поверхні зовнішнього носа, верхньої губи. Лікар при огляді встановлює запалення другої гілки трійчастого нерва. Через який отвір виходить із черепа ця гілка? A 54-year-old man was admitted to the neurosurgery department with complaints of lack of sensitivity of the skin of the lower eyelid, the lateral surface of the external nose, and the upper lip. During the examination, the doctor found inflammation of the second branch of the trigeminal nerve. Through which hole does this branch leave the skull?

Круглий отвір Round hole

Остистий отвір Prickly Hole

Рваний отвір Torn hole

Овальний отвір Oval hole

Верхня очноямкова щілина Upper orbital fissure

41 / 200
Чоловіку 18-ти років з приводу флегмони плеча було зроблено внутрішньом’язову ін’єкцію пеніциліну. Після цього у нього з’явилися тахікардія, ниткоподібний пульс, АТ знизився до 80/60 мм рт.ст. Який вид фармакологічної реакції розвинувся? An 18-year-old man was given an intramuscular injection of penicillin due to phlegmon of the shoulder. After that, he developed tachycardia, a threadlike pulse, and blood pressure decreased up to 80/60 mm Hg. What type of pharmacological reaction developed?

Потенціювання Potentialization

Центральна дія Central Action

Периферична дія Peripheral action

Рефлекторна дія Reflex action

Анафілаксія Anaphylaxis

42 / 200
Після побутової травми у пацієнта 18-ти років з’явились постійні запаморочення, ністагм очей, скандована мова, невпевнена хода. Це свідчить про порушення функції: After a domestic injury, an 18-year-old patient developed constant dizziness, eye nystagmus, slurred speech, and unsteady gait. This indicates a functional impairment:

Мозочка Cerebellum

Чорної субстанції Black substance

Вестибулярних ядер Vestibular nuclei

Рухової кори Motor cortex

Базальних гангліїв Basal ganglia

43 / 200
Хворий 65-ти років, що страждає на атеросклероз, госпіталізований до хірургічного відділення з приводу розлитого гнійного перитоніту. Під час операції діагностовано тромбоз брижових артерій. Яка найбільш імовірна причина перитоніту? A 65-year-old patient suffering from atherosclerosis was admitted to the surgical department for diffuse purulent peritonitis. During the operation, thrombosis of the mesenteric arteries was diagnosed. What is the most likely cause peritonitis?

Ішемія компресійна Compression ischemia

Геморагічний інфаркт Hemorrhagic heart attack

Стаз Stage

Ішемічний інфаркт Ischemic heart attack

Ішемія ангіоспастична Angiospastic ischemia

44 / 200
Хворому з прогресуючою м’язовою дистрофією було проведено біохімічне дослідження сечі. Поява якої речовини у великій кількості в сечі може підтвердити захворювання м’язів у даного хворого? A patient with progressive muscular dystrophy underwent a biochemical examination of urine. The appearance of a substance in large quantities in the urine can confirm muscle disease in this patient?

Гіпурова кислота Hippuric acid

Креатинін Creatinine

Порфірини Porphyrins

Креатин Creatine

Сечовина Urea

45 / 200
У крові чоловіка 26-ти років виявлено 18% еритроцитів сферичної, сплощеної, кулястої та остистої форм. Інші еритроцити були у формі двоввігнутих дисків. Як називається таке явище? In the blood of a 26-year-old man, 18% of erythrocytes were found to be spherical, flattened, spherical and spiny. Other erythrocytes were in the form of biconcave discs. What is this phenomenon called?

Еритроцитоз Erythrocytosis

Фізіологічний пойкілоцитоз Physiological poikilocytosis

Патологічний анізоцитоз Pathological anisocytosis

Фізіологічний анізоцитоз Physiological anisocytosis

Патологічний пойкілоцитоз Pathological poikilocytosis

46 / 200
Хвора страждає від болю в ногах та набряків. В ході обстеження хворої на медіальній поверхні стегна спостерігається набряк, збільшення розміру вен, утворення вузлів. З боку якої вени спостерігається патологія? The patient suffers from pain in the legs and swelling. During the examination of the patient on the medial surface of the thigh, there is swelling, an increase in the size of the veins, and the formation of nodes. On which side of the vein is the pathology observed ?

V femoralis V femoralis

V tibialis V tibialis

V saphena magna V saphena magna

V profunda femoris V profunda femoris

V saphena parva V saphena parva

47 / 200
Під час підготовки пацієнта до операції на серці проведено вимірювання тиску в камерах серця. В одній з них тиск протягом серцевого циклу змінювався від 0 до 120 мм рт.ст. Назвіть цю камеру серця: During the patient's preparation for heart surgery, pressure was measured in the chambers of the heart. In one of them, the pressure during the cardiac cycle varied from 0 to 120 mmHg. Name this chamber of the heart:

Праве передсердя Right atrium

Лівий шлуночок Left Ventricle

Правий шлуночок Right ventricle

- -

Ліве передсердя Left atrium

48 / 200
У жінки обмежений кровотік у нирках, підвищений артеріальний тиск. Гіперсекреція якого гормону зумовила підвищений тиск? The woman has limited blood flow in the kidneys, high blood pressure. Hypersecretion of which hormone caused the high pressure?

Вазопресин Vasopressin

Адреналін Adrenaline

Еритропоетин Erythropoietin

Норадреналін Noradrenaline

Ренін Renin

49 / 200
На останньому місяці вагітності вміст фібриногену в плазмі крові в 2 рази вище за норму. Яку швидкість осідання еритроцитів слід при цьому очікувати? In the last month of pregnancy, the content of fibrinogen in the blood plasma is 2 times higher than the norm. What should be the rate of erythrocyte sedimentation?

0-5 мм/годину 0-5 mm/hour

40-50 мм/годину 40-50 mm/hour

10-15 мм/годину 10-15 mm/hour

5-10 мм/годину 5-10 mm/hour

3-12 мм/годину 3-12 mm/hour

50 / 200
Пацієнт 16-ти років, що страждає на хворобу Іценко-Кушінга, консультований з приводу надмірної ваги тіла. При опитуванні з’ясувалося, що енергетична цінність спожитої їжі складає 1700-1900 ккал/добу. Яка провідна причина ожиріння у даному випадку? A 16-year-old patient suffering from Itsenko-Cushing's disease is consulted about excess body weight. The survey revealed that the energy value of the consumed food is 1700-1900 kcal/day. What is the leading cause of obesity in this case?

Нестача інсуліну Lack of insulin

Нестача глюкокортикоїдів Lack of glucocorticoids

Надлишок глюкокортикоїдів Glucocorticoid excess

Гіподинамія Hypodynamia

Надлишок інсуліну Excess insulin

51 / 200
Людина зробила спокійний видих. Як називається об’єм повітря, який міститься у неї в легенях при цьому? A person exhaled calmly. What is the volume of air contained in his lungs during this called?

Резервний об’єм видиху Reserve expiratory volume

Функціональна залишкова ємкість легень Functional residual lung capacity

Залишковий об’єм Remaining Volume

Дихальний об’єм Respiratory volume

Життєва ємність легень Vital lung capacity

52 / 200
Людина зробила максимально глибокий видих. Як називається об’єм повітря, що знаходиться в її легенях після цього? A person exhaled as deeply as possible. What is the volume of air in his lungs after that called?

Залишковий об’єм Remaining Volume

Резервний об’єм видиху Reserve expiratory volume

Ємність вдиху Inhalation capacity

Функціональна залишкова ємність легень Functional residual lung capacity

Альвеолярний об’єм Alveolar volume

53 / 200
В експерименті на ссавці зруйнували певну структуру серця, що призвело до припинення проведення збудження від передсердь до шлуночків. Що саме зруйнували? In an experiment on a mammal, a certain structure of the heart was destroyed, which led to the cessation of conduction of excitation from the atria to the ventricles. What exactly was destroyed?

Атріовентрикулярний вузол Atrioventricular node

Ніжки пучка Гіса His Bundle Legs

Волокна Пуркін’є Purkinje fibers

Синоатріальний вузол Sinoatrial node

Пучок Гіса His Bundle

54 / 200
Необхідно оцінити рівень збудливості нерва у хворого. Для цього доцільно визначити для нерва наступну величину: It is necessary to assess the level of nerve excitability in the patient. For this, it is advisable to determine the following value for the nerve:

Порогова сила подразника Threshold stimulus strength

Амплітуда потенціалу дії Action potential amplitude

Критичний рівень деполяризації Critical level of depolarization

Тривалість потенціалу дії Action potential duration

Потенціал спокою Rest potential

55 / 200
Хворому з гострою недостатністю надниркових залоз був призначений лікарський препарат після якого у нього з’явилися скарги на біль в кістках (двічі були переломи), часті простудні хвороби, набряки, повільне загоювання ран. Який препарат міг спричинити такі явища? A patient with acute adrenal insufficiency was prescribed a drug, after which he complained of pain in the bones (twice had fractures), frequent colds, swelling , slow wound healing. What drug could cause such phenomena?

Спіронолактон Spironolactone

Тестостерон Testosterone

Преднізолон Prednisone

Ретаболіл Retabolil

Естріол Estriol

56 / 200
Під час розтину тіла мертвонародженої дитини виявлено аномалію розвитку серця: шлуночки не розмежовані, з правої частини виходить суцільний артеріальний стовбур. Для яких хребетних характерна подібна будова серця? During the autopsy of the body of a stillborn child, an abnormality in the development of the heart was found: the ventricles are not demarcated, a solid arterial trunk emerges from the right part. What vertebrates have a similar structure of the heart?

Рептилії Reptiles

Амфібії Amphibia

Ссавці Mammals

Птахи Birds

Риби Fish

57 / 200
Який з нижченаведених сечогінних засобів слід призначити хворому з первинним гіперальдостеронізмом? Which of the following diuretics should be prescribed to a patient with primary hyperaldosteronism?

Триамтерен Triamterene

Фуросемід Furosemide

Гіпотіазид Hypotiazid

Маніт Manit

Спіронолактон Spironolactone

58 / 200
В результаті травми хворий не може підняти руку до горизонтального рівня. Який м’яз постраждав? As a result of the injury, the patient cannot raise his arm to the horizontal level. What muscle is affected?

Дельтоподібний Delta

Трапецієподібний Trapezoid

Триголовий Three-headed

Двоголовий Two-headed

Найширший м’яз спини Latisus dorsi

59 / 200
В експерименті збільшили проникність мембрани збудливої клітини для іонів калію. Які зміни електричного стану мембрани при цьому виникнуть? In the experiment, the permeability of the excitable cell membrane for potassium ions was increased. What changes in the electrical state of the membrane will occur?

Локальна відповідь Local response

Змін не буде There will be no changes

Потенціал дії Action potential

Деполяризація Depolarization

Гіперполяризація Hyperpolarization

60 / 200
При обстеженні в аналізі крові пацієнта виявлено лейкоцитоз, лімфоцитоз, клітини Боткіна- Гумпрехта на тлі анемії. Про яку хворобу слід думати лікарю? When examining the patient's blood, leukocytosis, lymphocytosis, Botkin-Gumprecht cells against the background of anemia were detected. What disease should the doctor think about?

Мієломна хвороба Myeloma

Інфекційний мононуклеоз Infectious mononucleosis

Гострий мієлолейкоз Acute myelogenous leukemia

Лімфогранулематоз Lymphogranulomatosis

Хронічний лімфолейкоз Chronic lymphocytic leukemia

61 / 200
У жінки 37-ми років протягом року періодично виникали інфекційні захворювання бактеріального генезу, їх перебіг був вкрай тривалим, ремісії - короткочасними. При обстеженні виявлена гіпогамаглобулінемія. Порушення функції яких клітин може бути прямою її причиною? A 37-year-old woman periodically developed infectious diseases of bacterial origin during the year, their course was extremely long, remissions were short-lived. During the examination, hypogammaglobulinemia was detected. Violations of the functions of cells can be its direct cause?

Плазматичні клітини Plasma cells

Нейтрофіли Neutrophils

Лімфоцити Lymphocytes

Фагоцити Phagocytes

Макрофаги Macrophages

62 / 200
У хворого 37-ми років на фоні тривалого застосування антибіотиків спостерігається підвищена кровоточивість при невеликих пошкодженнях. У крові - зниження активності факторів згортання крові ІІ, VII, IX, X, подовження часу згортання крові. Недостатністю якого вітаміну обумовлені зазначені зміни? In a 37-year-old patient, against the background of long-term use of antibiotics, there is increased bleeding with small injuries. In the blood, there is a decrease in the activity of blood coagulation factors II, VII, IX, X , lengthening of the blood clotting time. The deficiency of which vitamin is responsible for these changes?

Вітамін С Vitamin C

Вітамін D Vitamin D

Вітамін Е Vitamin E

Вітамін К Vitamin K

Вітамін А Vitamin A

63 / 200
Хворий 18-ти років звернувся до лікарні із скаргами на шум та больові відчуття у вусі. Об’єктивно - у хворого гостре респіраторне захворювання, риніт. Крізь який отвір глотки інфекція потрапила до барабанної порожнини та викликала її запалення? An 18-year-old patient came to the hospital with complaints of noise and pain in the ear. Objectively, the patient has an acute respiratory disease, rhinitis. Through which opening pharynx infection got to the tympanic cavity and caused its inflammation?

Хоани Hoans

Зів Ziv

Глотковий отвір слухової труби Pharyngeal opening of the auditory tube

Барабанний отвір слухової труби Tympanic opening of the auditory tube

Вхід до гортані Entrance to the larynx

64 / 200
У хворого 68-ми років, який страждає на серцеву недостатність та впродовж тривалого часу приймає препарати наперстянки, з’явилися явища інтоксикації, які швидко нівелювалися застосуванням донатора сульфгідрильних груп унітіолу. Який механізм терапевтичної дії цього засобу? In a 68-year-old patient who suffers from heart failure and has been taking digitalis drugs for a long time, intoxication symptoms appeared, which were quickly eliminated by the use of a donor of sulfhydryl groups unitiol. What is the mechanism of therapeutic action of this agent?

Сповільнює надходження натрію до міокардіоцитів Slows sodium intake to myocardiocytes

Реактивує натрій-калієву-АТФ-азу мембран міокардіоцитів Reactivates sodium-potassium-ATP-ase of myocardiocyte membranes

Іальмує вивільнення калію з міокардіоцитів Increases potassium release from myocardiocytes

Підвищує енергозабезпечення міокарду Increases the energy supply of the myocardium

Зменшує накопичення іонізованого кальцію Reduces the accumulation of ionized calcium

65 / 200
Охолодження тіла людини у воді виникає значно швидше, ніж на повітрі, тому, що у воді значно ефективнішою є віддача тепла шляхом: Cooling of the human body occurs much faster in water than in air, because heat transfer is much more efficient in water by:

- -

Тепловипромінювання Heat radiation

Конвекції Convection

Випаровування поту Evaporation of sweat

Теплопроведення Heat conduction

66 / 200
У бактеріологічній лабораторії проводиться дослідження якості питної води. Її мікробне число виявилося близько 100. Які мікроорганізми враховувалися при цьому? The quality of drinking water is being tested in the bacteriological laboratory. Its microbial count was found to be about 100. What microorganisms were taken into account?

Бактерії, патогенні для людей та тварин Bacteria pathogenic for humans and animals

Умовно-патогенні мікроорганізми Conditionally pathogenic microorganisms

Всі бактерії, що виросли на живильному середовищі All bacteria grown on nutrient medium

Ентеропатогенні бактерії та віруси Enteropathogenic bacteria and viruses

Бактерії групи кишкової палички Escherichia coli bacteria

67 / 200
У юнака 20-ти років діагностовано спадковий дефіцит УДФ-глюкуронілтрансферази. Підвищення якого показника крові підтверджує діагноз? A 20-year-old man was diagnosed with a hereditary deficiency of UDP-glucuronyltransferase. An increase in which blood level confirms the diagnosis?

Уробілін Urobilin

Тваринний індикан Animal Indican

Прямий (кон’югований) білірубін Direct (conjugated) bilirubin

Непрямий (некон’югований) білірубін Indirect (unconjugated) bilirubin

Стеркобіліноген Stercobilinogen

68 / 200
Хворому 50-ти років з хронічною серцевою недостатністю і тахиаритмією призначили кардіотонічний препарат. Який з препаратів призначили хворому? A 50-year-old patient with chronic heart failure and tachyarrhythmia was prescribed a cardiotonic drug. Which drug was prescribed to the patient?

Мілдронат Mildronate

Добутамін Dobutamine

Дофамін Dopamine

Аміодарон Amiodarone

Дигоксин Digoxin

69 / 200
У людини порушено всмоктування продуктів гідролізу жирів. Причиною цього може бути дефіцит у порожнині тонкої кишки: A person has impaired absorption of fat hydrolysis products. The reason for this may be a deficiency in the cavity of the small intestine:

Ліполітичних ферментів Lipolytic enzymes

Жиророзчинних вітамінів Fat-soluble vitamins

Іонів натрію Sodium ions

Жовчних кислот Bile acids

Жовчних пігментів Bile pigments

70 / 200
У відповідь на сильне швидке скорочення м’яза спостерігається його рефлекторне розслаблення. З подразнення яких рецепторів починається ця рефлекторна реакція? In response to a strong rapid contraction of a muscle, its reflex relaxation is observed. With the irritation of which receptors does this reflex reaction begin?

М’язові веретена Muscle spindles

Суглобові рецептори Joint receptors

Больові рецептори Pain receptors

Дотикові рецептори Touch receptors

Сухожилкові рецептори Гольджі Golgi tendon receptors

71 / 200
У хворого, що переніс 5 років тому субтотальну резекцію шлунка, розвинулась В12-фолієводефіцитна анемія. Який механізм є провідним у розвитку такої анемії? A patient who underwent a subtotal resection of the stomach 5 years ago developed B12-folate-deficient anemia. What is the leading mechanism in the development of such anemia?

Порушення всмоктування вітаміну В в тонкій кишці Disturbance of vitamin B absorption in the small intestine

Дефіцит транскобаламіну Transcobalamin deficiency

Відсутність зовнішнього фактора Касла Absence of external Castle factor

Відсутність внутрішнього фактора Касла No internal Castle factor

Дефіцит фолієвої кислоти Folic acid deficiency

72 / 200
Чоловік 50-ти років хворіє на хронічний бронхіт, скаржиться на задишку під час фізичного навантаження, постійний кашель з відходженням харкотиння. При обстеженні діагностовано ускладнення - емфізема легень. Чим вона зумовлена? A 50-year-old man suffers from chronic bronchitis, complains of shortness of breath during physical exertion, constant cough with expectoration. During the examination, a complication was diagnosed - emphysema of the lungs. What is it conditioned?

Зменшення перфузії легень Decreased lung perfusion

Зниження еластичних властивостей легень Decreasing the elastic properties of the lungs

Порушення вентиляційно-перфузійного співвідношення в легенях Violation of the ventilation-perfusion ratio in the lungs

Зменшення розтяжності легень Decreased lung distensibility

Зменшення альвеолярної вентиляції Decreased alveolar ventilation

73 / 200
При розтині трупа чоловіка 47-ми років, померлого раптово, в інтимі черевного відділу аорти знайдені осередки жовтого кольору у вигляді плям та смуг, що не вибухають над поверхнею інтими. При фарбуванні суданом III спостерігається жовтогаряче забарвлення. Для якої стадії атеросклерозу характерні такі зміни? At the autopsy of a 47-year-old man who died suddenly, yellow cells were found in the intima of the abdominal part of the aorta in the form of spots and stripes that did not explode above the intima surface When stained with Sudan III, a yellow-red color is observed. What stage of atherosclerosis is characterized by such changes?

Ліпоїдозу Lipoidosis

Стадія утворення атероматозної виразки Stage of formation of atheromatous ulcer

Атероматозу Atheromatosis

Ліпосклерозу Liposclerosis

Атерокальцинозу Atherocalcinosis

74 / 200
У клініку госпіталізовано хворого з діагнозом карциноїду кишечника. Аналіз виявив підвищену продукцію серотоніну, який утворюється з амінокислоти триптофан. Який біохімічний механізм лежить в основі даного процесу? A patient with a diagnosis of intestinal carcinoid was hospitalized in the clinic. The analysis revealed an increased production of serotonin, which is formed from the amino acid tryptophan. What biochemical mechanism is the basis of this process?

Декарбоксилювання Decarboxylation

Утворення парних сполук Formation of paired compounds

Мікросомальне окиснення Microsomal oxidation

Трансамінування Transamination

Дезамінування Demining

75 / 200
У батьків, хворих на гемоглобінопатію (аутосомно-домінантний тип успадкування), народилася здорова дівчинка. Які генотипи батьків? Parents suffering from hemoglobinopathy (autosomal dominant type of inheritance) had a healthy girl. What are the parents' genotypes?

Обоє гомозиготні за геном гемоглобінопатії Both are homozygous for the hemoglobinopathy gene

Обоє гетерозиготні за геном гемоглобінопатії Both are heterozygous for the hemoglobinopathy gene

Мати гетерозиготна за геном гемоглобінопатії, у батька цей ген відсутній The mother is heterozygous for the hemoglobinopathy gene, the father lacks this gene

У обох батьків ген гемоглобінопатії відсутній Both parents have no hemoglobinopathy gene

Батько гетерозиготний за геном гемоглобінопатії, у матері цей ген відсутній The father is heterozygous for the hemoglobinopathy gene, the mother does not have this gene

76 / 200
Хворому з артеріальною гіпертензією було призначено один з антигіпертензивних засобів. Артеріальний тиск нормалізувався, однак хворого почав турбувати постійний сухий кашель. Який з перерахованих препаратів має таку побічну дію? A patient with arterial hypertension was prescribed one of the antihypertensive drugs. Blood pressure normalized, but the patient began to be bothered by a constant dry cough. Which of the listed drugs has such a side effect?

Анаприлін Anaprilin

Лізиноприл Lisinopril

Клофелін Clofelin

Резерпін Reserpin

Ніфедипін Nifedipine

77 / 200
В анотації до препарату вказано, що він містить антигени збудника черевного тифу, адсорбовані на стабілізованих еритроцитах барана. З якою метою використовують цей препарат? In the annotation to the drug, it is indicated that it contains antigens of the causative agent of typhoid, adsorbed on stabilized erythrocytes of a ram. For what purpose is this drug used?

Для виявлення антитіл в реакції Відаля To detect antibodies in the Vidal reaction

Для виявлення антитіл в реакції гальмування гемаглютинації To detect antibodies in the hemagglutination inhibition reaction

Для серологічної ідентифікації збудника черевного тифу For serological identification of the causative agent of typhoid fever

Для виявлення антитіл в реакції зв’язування комплементу To detect antibodies in the complement binding reaction

Для виявлення антитіл в реакції непрямої гемаглютинації To detect antibodies in the indirect hemagglutination reaction

78 / 200
При розтині тіла померлого чоловіка 48-ми років в ділянці 1-го сегменту правої легені виявлено круглий утвір діаметром 5 см з чіткими контурами, оточений тонким прошарком сполучної тканини, виповнений білими крихкими масами. Діагностуйте форму вторинного туберкульозу: At the autopsy of the body of a deceased 48-year-old man, a round formation with a diameter of 5 cm with clear contours, surrounded by a thin layer of connective tissue, was found in the area of the 1st segment of the right lung. filled with white friable masses. Diagnose the form of secondary tuberculosis:

Гострий вогнищевий туберкульоз Acute focal tuberculosis

Гострий кавернозний туберкульоз Acute cavernous tuberculosis

Фіброзно-кавернозний туберкульоз Fibro-cavernous tuberculosis

Казеозна пневмонія Case pneumonia

Туберкулома Tuberculosis

79 / 200
У хворого діагностовано септичний ендокардит. Температура тіла протягом 5-ти днів коливалася в межах 39,5oC -40,2oC. На 6-й день на тлі різкого зниження температури до35,2oC розвинувся колапс. Який головний механізм колапсу? The patient was diagnosed with septic endocarditis. The body temperature fluctuated between 39.5oC and 40.2oC for 5 days. On the 6th day, against the background of a sharp decrease at temperatures up to 35.2oC, collapse developed. What is the main mechanism of collapse?

Поліурія Polyuria

Посилене потовиділення Increased sweating

Гіпервентиляція Hyperventilation

Вазодилатація Vasodilation

Тахікардія Tachycardia

80 / 200
Хвора звернулася зі скаргами на болі у правій латеральній ділянці живота. Під час пальпації визначається щільне, нерухоме, пухлиноподібне утворення. У ділянці якого відділу травної трубки можлива наявність пухлини? The patient complained of pain in the right lateral part of the abdomen. During palpation, a dense, immobile, tumor-like formation is detected. In which part of the digestive tube is the presence of a tumor possible?'

Colon ascendens Colon ascendens

Colon descendens Colon descendens

Colon transversum Colon transversum

Colon sigmoideum Colon sigmoideum

Caecum Caecum

81 / 200
У хворого струс головного мозку, що супроводжується повторним блюванням і задишкою. При обстеженні відзначено: рН - 7,62; pCO - 40 мм рт.ст. Яке порушення кислотно-основного стану є у хворого? The patient has a brain concussion, accompanied by repeated vomiting and shortness of breath. During the examination, it was noted: pH - 7.62; pCO - 40 mm Hg. What a violation Does the patient have an acid-base state?

- -

Негазовий алкалоз Nongaseous alkalosis

Газовий алкалоз Gas alkalosis

Газовий ацидоз Gas acidosis

Негазовий ацидоз Nongaseous acidosis

82 / 200
У хворого в обох щелепах рентгенологічно виявлено численні дефекти у вигляді гладкостінних округлих отворів. При гістологічному дослідженні - явища остеолізису і остеопорозу при явищах слабкого кісткоутворення. В сечі хворого знайдено білок Бенс-Джонса. Назвіть захворювання: In both jaws of the patient, numerous defects in the form of smooth-walled round holes were detected x-ray. Histological examination revealed the phenomena of osteolysis and osteoporosis with weak bone formation. Protein was found in the patient's urine Bence-Jones. Name the disease:

Хронічний мієлолейкоз Chronic myelogenous leukemia

Хронічний еритромієлоз Chronic erythromyelosis

Мієломна хвороба Myeloma

Гострий недиференційований лейкоз Acute undifferentiated leukemia

Гострий мієлолейкоз Acute myelogenous leukemia

83 / 200
Катіонні глікопротеїни є основними компонентами слини привушних залоз. Які амінокислоти обумовлюють їх позитивний заряд? Cationic glycoproteins are the main components of parotid gland saliva. What amino acids determine their positive charge?

Лізин, аргінін, гістидин Lysine, arginine, histidine

Аспартат, аргінін, глутамат Aspartate, arginine, glutamate

Глутамат, валін, лейцин Glutamate, valine, leucine

Аспартат, глутамат, гліцин Aspartate, glutamate, glycine

Цистеїн, гліцин, пролін Cysteine, glycine, proline

84 / 200
На мікропрепараті очного яблука плода спостерігається пошкодження рогівки. Частина якого зародкового листка була уражена в процесі ембріонального розвитку? Cornea damage is observed on the micro-preparation of the eyeball of the fetus. Which part of the germ layer was affected during embryonic development?

Мезодерма Mesoderm

Дерматом Dermatoma

Нефротом Nephrotoma

Ентодерма Entoderm

Ектодерма Ectoderm

85 / 200
На практичному занятті з мікробіології студентам запропоновано пофарбувати суміш бактерій за методикою Грама та пояснити механізм фарбування. Які морфологічні структури бактерій зумовлюють грамнегативне та грампозитивне фарбування бактерій? In a practical lesson in microbiology, students were asked to stain a mixture of bacteria using the Gram method and explain the staining mechanism. What morphological structures of bacteria cause gram-negative and gram-positive staining of bacteria?

Капсула Capsule

Джгутики Flags

ЦПМ CPM

Цитоплазма Cytoplasm

Клітинна стінка Cell wall

86 / 200
У вагітної жінки взяли кров для підтвердження клінічного діагнозу 'токсоплазмоз' Яка з перерахованих серологічних реакцій має діагностичне значення? Blood was taken from a pregnant woman to confirm the clinical diagnosis of 'toxoplasmosis'. Which of the listed serological reactions has a diagnostic value?

Реакція гальмування гемаглютинації Hemagglutination inhibition reaction

Реакція гемадсорбції Hemadsorption reaction

Реакція аглютинації Agglutination reaction

Реакція зв’язування комплементу Complement binding reaction

Реакція нейтралізації Neutralization reaction

87 / 200
У здорових батьків, спадковість яких не обтяжена, народилася дитина з множинними вадами розвитку. Цитогенетичний аналіз виявив у соматичних клітинах дитини трисомію за 18-ю хромосомою (синдром Едвардса). З яким явищем пов’язане народження такої дитини? A child with multiple developmental disabilities was born to healthy parents whose heredity is not burdened. Cytogenetic analysis revealed trisomy on the 18th chromosome (Edwards syndrome) in the child's somatic cells What phenomenon is associated with the birth of such a child?

Хромосомною мутацією - дуплікацією By chromosomal mutation - duplication

Нерозходженням пари хромосом під час гаметогенезу Non-separation of a pair of chromosomes during gametogenesis

Соматичною мутацією у ембріона By somatic mutation in the embryo

Впливом тератогенних факторів Influence of teratogenic factors

Домінантною мутацією By dominant mutation

88 / 200
На препараті представлено орган, вкритий сполучнотканинною капсулою, від якої відходять трабекули. В органі можна розрізнити кіркову речовину, де містяться лімфатичні вузлики та мозкову речовину, представлену тяжами лімфоїдних клітин. Який орган представлений на препараті? The preparation shows an organ covered by a connective tissue capsule from which trabeculae depart. In the organ, you can distinguish the cortical substance, which contains lymph nodes, and the medullary substance, represented by strands of lymphoid cells What organ is represented on the drug?

Мигдалики Tonsils

Лімфатичний вузол Lymph node

Червоний кістковий мозок Red bone marrow

Селезінка Spleen

Тимус Thymus

89 / 200
При аналізі родоводу пробанда виявлено, що ознака проявляється з однаковою частотою у представників обох статей і хворі наявні у всіх поколіннях (по вертикалі), а по горизонталі - у сибсів (братів і сестер пробанда) з відносно великих родин. Який тип успадкування досліджуваної ознаки? When analyzing the pedigree of the proband, it was found that the trait appears with the same frequency in representatives of both sexes and patients are present in all generations (vertically), and horizontally - in sibs (siblings of the proband) from relatively large families. What is the type of inheritance of the trait under study?

Зчеплений з Х-хромосомою, рецесивний X-linked recessive

Автосомно-домінантний Autosomal dominant

Зчеплений з Х-хромосомою, домінантний X-linked, dominant

Зчеплений з Y-хромосомою Linked to Y-chromosome

Автосомно-рецесивний Autosomal recessive

90 / 200
У 60-річного пацієнта було виявлено гіперглікемію і глюкозурію. Для лікування хворого лікар призначив препарат для приймання всередину. Який це препарат? A 60-year-old patient was diagnosed with hyperglycemia and glucosuria. To treat the patient, the doctor prescribed a drug to be taken orally. What is this drug?

Панкреатин Pancreatin

Корглікон Corglycon

Глібенкламід Glibenclamide

Фуросемід Furosemide

Окситоцин Oxytocin

91 / 200
Хворий 58-ми років помер від прогресуючої серцевої недостатності. На розтині: серце розширене у поперечнику, мляве, м’яз на розрізі нерівномірного кровонаповнення, пістрявий. При гістологічному дослідженні: у міокарді повнокров’я, у стромі лімфогістіоцитарні інфільтрати, що розсувають кардіоміоцити. Виявлені морфологічні зміни свідчать про: A 58-year-old patient died of progressive heart failure. At autopsy: the heart is dilated in cross section, lethargic, the muscle on the section has uneven blood supply, speckled. At histological in the study: in the myocardium there is blood, in the stroma there are lymphohistiocytic infiltrates, displacing cardiomyocytes. The detected morphological changes indicate:

Негнійний проміжний міокардит Nonpurulent interstitial myocarditis

Кардіосклероз Cardiosclerosis

Венозне повнокров’я Venous complete blood

Жирову дистрофію міокарда Fatty dystrophy of the myocardium

Інфаркт міокарда Myocardial infarction

92 / 200
У важкоатлета при підйомі штанги відбувся розрив грудної лімфатичної протоки. Вкажіть найбільш імовірне місце ушкодження: A weightlifter ruptured a thoracic lymphatic duct while lifting a barbell. Specify the most likely location of the injury:

Ділянка шиї Neck Area

Місце впадіння у венозний кут Place of confluence with the venous angle

Ділянка аортального отвору діафрагми Area of the aortic orifice of the diaphragm

Заднє середостіння Posterior mediastinum

Ділянка попереково-крижового сполучення Section of lumbar-sacral connection

93 / 200
При пошкодженні клітини іонізуючим випромінюванням вмикаються механізми захисту і адаптації. Який механізм відновлення порушеного внутрішньоклітинного гомеостазу реалізується при цьому? When a cell is damaged by ionizing radiation, protection and adaptation mechanisms are activated. What mechanism of restoration of disturbed intracellular homeostasis is implemented in this case?

Пригнічення аденілатциклази Inhibition of adenylate cyclase

Активація Са-опосередкованих клітинних функцій Activation of Ca-mediated cellular functions

Активація антиоксидантної системи Activation of the antioxidant system

Накопичення Na+ в клітинах Na+ accumulation in cells

Гіпертрофія мітохондрій Mitochondrial hypertrophy

94 / 200
У хворої після видалення матки розвинулась гостра анурiя (немає виділення сечі). Які анатомiчнi структури найімовірніше було пошкоджено при операції? After removing the uterus, the patient developed acute anuria (no urine output). What anatomical structures were most likely damaged during the operation?

Сечівник Urine

Зовнішній сфінктер сечівника External urethral sphincter

Цибулинно-губчастий м’яз Onion spongy muscle

Внутрішній сфінктер сечівника Internal urethral sphincter

Сечоводи Ureters

95 / 200
У хворої внаслідок запалення порушена ендокринна функція фолікулярних клітин фолікулів яєчника. Синтез яких гормонів буде пригнічений? The endocrine function of the follicular cells of the ovarian follicles is disturbed in the patient as a result of inflammation. The synthesis of which hormones will be inhibited?

Лютропін Lutropin

Естрогени Estrogens

Фолістатин Follistatin

Прогестерон Progesterone

Фолікулостимулюючий гормон Follicle-stimulating hormone

96 / 200
У чоловіка при обстеженні виявлено порушення кровообігу міокарда лівого передсердя. У басейні якої артерії відбулись порушення кровообігу? During the examination, a man was found to have a violation of the blood circulation of the left atrial myocardium. In the basin of which artery did the blood circulation violations occur?

Права вінцева Coronal right

Ліва вінцева Left coronal

Права та ліва вінцеві Right and left corona

- -

Передня міжшлуночкова гілка лівої вінцевої артерії Anterior interventricular branch of the left coronary artery

97 / 200
Чоловік 36-ти років, лісник за фахом, через тиждень після тривалого перебування у весняному лісі гостро захворів - гарячка, головний біль, порушення свідомості,епілептиформні напади, смерть розвинулася на 3-й день хвороби. На розтині тіла: набрякголовного мозку, множинні точкові геморагії; під час мікроскопічного дослідження - периваскулярний та перицелюлярний набряк, множинні периваскулярні, переважно лімфоцитарні, інфільтрати. Діагностуйте основне захворювання: A 36-year-old man, a forester by profession, became acutely ill a week after a long stay in the spring forest - fever, headache, loss of consciousness, epileptiform seizures, death developed on the 3rd day of illness. On autopsy: cerebral edema, multiple point hemorrhages; during microscopic examination - perivascular and pericellular edema, multiple perivascular, mainly lymphocytic, infiltrates. Diagnose the main disease:

Гнійний енцефаліт Suppurative encephalitis

Кліщовий енцефаліт Tick-borne encephalitis

Поліомієліт Polio

Церебро-васкулярна хвороба Cerebrovascular disease

Менінгококова інфекція Meningococcal infection

98 / 200
При зовнішньому дослідженні трупа чоловіка 69-ти років, який помер 4 години тому, патологоанатом відмітив, що м’язи померлого мають дуже щільну консистенцію, суглоби згинаються та розгинаються важко. Як називається ця патологоанатомічна ознака смерті? During the external examination of the corpse of a 69-year-old man who died 4 hours ago, the pathologist noted that the muscles of the deceased have a very dense consistency, the joints bend and flex difficult. What is the name of this pathological sign of death?

Трупне заклякання Corpse Charm

Трупне охолодження Corporal cooling

Трупне розкладання Decomposition

Трупні гіпостази Corpse Hypostases

Трупне висихання Corpse desiccation

99 / 200
Хворому з метою попередження жирової дистрофії печінки лікар призначив ліпотропний препарат - донор метильних груп. Це імовірно: In order to prevent fatty liver dystrophy, the doctor prescribed a lipotropic drug - a donor of methyl groups. This is likely:

S-Аденозилметіонін S-Adenosylmethionine

Холестерин Cholesterol

Білірубін Bilirubin

Глюкоза Glucose

Валін Valin

100 / 200
Хворому на гострий інфаркт міокарда у комплексній терапії було призначено гепарин. Через деякий час після введення даного препарату з’явилась гематурія. Який антагоніст гепарину необхідно ввести хворому для усунення даного ускладнення? A patient with an acute myocardial infarction was prescribed heparin in complex therapy. Some time after the administration of this drug, hematuria appeared. What heparin antagonist should be administered to the patient to eliminate this complication?

Вікасол Vikasol

Амінокапронова кислота Aminocaproic acid

Неодикумарин Neodicoumarin

Протаміну сульфат Protamine sulfate

Фібриноген Fibrinogen

101 / 200
У хворого виявили злоякісну анемію. Терапія внутрішньо-м’язовим введення вітаміну Б12 давала нетривалий нестійкий ефект поліпшення складу крові. Пацієнт - завзятий рибаль і часто вживає самостійно виловлену і недостатньо термічно оброблену рибу. Який діагноз можна припустити? The patient was diagnosed with pernicious anemia. Therapy with intramuscular administration of vitamin B12 gave a short-term, unstable effect of improving blood composition. The patient is an avid fisherman and often consumes self-caught and undercooked fish. What diagnosis can be assumed?

Парагонімоз Paragonimosis

Анкілостомоз Ankylostomosis

Трихоцефальоз Trichocephalus

Ентеробіоз Enterobiosis

Дифілоботріоз Diphyllobotriosis

102 / 200
У синтезі пуринових нуклеотидів беруть участь деякі амінокислоти, похідні вітамінів, фосфорні ефіри рибози. Коферментна форма якого вітаміну є переносником одновуглецевих фрагментів в цьому синтезі? In the synthesis of purine nucleotides, some amino acids, vitamin derivatives, phosphoric esters of ribose are involved. The coenzyme form of which vitamin is the carrier of one-carbon fragments in this synthesis?

Нікотинова кислота Nicotinic acid

Рибофлавін Riboflavin

Піридоксин Pyridoxine

Пантотенова кислота Pantothenic acid

Фолієва кислота Folic acid

103 / 200
У тварини зруйнували отолітові вестибулорецептори. Які з наведених рефлексів зникнуть внаслідок цього у тварини? The animal's otolith vestibuloreceptors were destroyed. Which of the following reflexes will disappear in the animal as a result?

Первинні орієнтувальні Primary indicative

Випрямлення тулуба Torso straightening

Статокінетичні при рухах з лінійним прискоренням Statokinetic when moving with linear acceleration

Міотатичні Myotatic

Статокінетичні при рухах з кутовим прискоренням Statokinetic during movements with angular acceleration

104 / 200
Лікар записав в історії хвороби, що у хворого дихання поверхневе (знижена глибина дихання). Це означає, що зменшеним є такий показник зовнішнього дихання: The doctor recorded in the medical history that the patient's breathing is shallow (reduced depth of breathing). This means that the external breathing rate is reduced:

Дихальний об’єм Respiratory volume

Життєва ємність легень Vital lung capacity

Функціональна залишкова ємність Functional residual capacity

Ємність вдиху Inhalation capacity

Хвилинний об’єм дихання Minute respiratory volume

105 / 200
До лікаря звернувся студент з проханням призначити препарат для лікування алергічного риніту, який виник у нього під час цвітіння липи. Який засіб можна застосувати? A student turned to the doctor with a request to prescribe a drug for the treatment of allergic rhinitis, which occurred during the flowering of the linden tree. What drug can be used?

Анаприлін Anaprilin

Лозартан Losartan

Норадреналіну гідротартрат Noradrenaline hydrotartrate

Лоратадин Loratadine

Амброксол Ambroxol

106 / 200
Хворий похилого віку страждає на хронічний закреп, в основі якого лежить гіпотонія товстої кишки. Який препарат слід призначити хворому? An elderly patient suffers from chronic constipation, the basis of which is hypotonia of the colon. What drug should be prescribed to the patient?

Новокаїнамід Novocaineamide

Бісакодил Bisacodyl

Натрію сульфат Sodium sulfate

Атропіну сульфат Atropine sulfate

Касторова олія Castor oil

107 / 200
При лабораторному дослідженні дитини виявлено підвищений вміст у крові та сечі лейцину, валіну, ізолейцину та їх кетопохідних. Сеча має характерний запах кленового сиропу. Недостатність якого ферменту характерно для цього захворювання? During the child's laboratory examination, an elevated content of leucine, valine, isoleucine and their keto derivatives was found in the blood and urine. The urine has a characteristic smell of maple syrup. The deficiency of which enzyme is characteristic of this disease?

Фосфофруктомутаза Phosphofructomutase

Дегідрогеназа розгалужених амінокислот Dehydrogenase of branched-chain amino acids

Амінотрансфераза Aminotransferase

Глюкозо-6-фосфатаза Glucose-6-phosphatase

Фосфофруктокіназа Phosphofructokinase

108 / 200
При обстеженні 2-х місячної дитини педіатр звернула увагу, що плач дитини нагадує котячий крик. Діагностовані мікроцефалія і вада серця. За допомогою цитогенетичного метода з’ясований каріотип дитини: 46, XX, 5р. Дане захворювання є наслідком такого процесу: During the examination of a 2-month-old child, the pediatrician noticed that the child's cry resembled a cat's cry. Microcephaly and a heart defect were diagnosed. The child's karyotype was determined using the cytogenetic method : 46, XX, 5yr. This disease is a consequence of the following process:

Плейотропія Pleiotropy

Інверсія Inversion

Дуплікація Duplication

Делеція Deletion

Транслокація Translocation

109 / 200
Під час аутопсії тіла хворого, котрий помер від тяжкого ексикозу, на ґрунті профузної діареї, виявлені такі зміни: слизова оболонка прямої та сигмоподібної кишок на всьому протязі вкрита сіро-білими плівчастими нашаруваннями, що міцно з’єднані з підлеглими тканинами, між плівками розташовані множинні великі й поверхневі виразки, вкриті згортками крові. Мікроскопічно діагностовано фібринозно-виразковий коліт. Яке захворювання проявляється такими змінами? During the autopsy of the body of a patient who died of severe exicosis, on the basis of profuse diarrhea, the following changes were found: the mucous membrane of the rectum and sigmoid intestine is covered with gray white membranous layers that are firmly connected to the underlying tissues, between the films there are multiple large and superficial ulcers covered with blood clots. Fibrinous-ulcerative colitis was diagnosed microscopically. What disease is manifested by such changes?

Колі-інфекція Coli infection

Ієрсініоз Yersiniosis

Дизентерія Dysentery

Стафілококова інфекція Staphylococcal infection

Сальмонельози Salmonellosis

110 / 200
У лікарню до кінця робочого дня доставлений працівник 'гарячого' цеху, який скаржиться на головний біль, запаморочення, нудоту, загальну слабкість. Свідомість збережена, шкірні покриви гіперемовані, сухі, гарячі на дотик. ЧСС- 130/хв. Дихання часте, поверхневе. Яке порушення процесів регуляції тепла найімовірніше виникло у людини у даній ситуації? By the end of the working day, an employee of a 'hot' workshop was taken to the hospital, complaining of headache, dizziness, nausea, general weakness. Consciousness is preserved, the skin is hyperemic, dry, hot to the touch. Heart rate - 130/min. Breathing is frequent, shallow. What violation of heat regulation processes most likely occurred in a person in this situation?

Посилення теплопродукції без змін тепловіддачі Increased heat production without changes in heat output

Зниження теплопродукції без зміни тепловіддачі Decreasing heat production without changing heat output

Зниження тепловіддачі Reduction of heat transfer

Посилення тепловіддачі і зниження теплопродукції Increasing heat transfer and decreasing heat production

Посилення тепловіддачі і теплопродукції Increasing heat transfer and heat production

111 / 200
В експерименті подразнюють гілочки симпатичного нерва, які іннервують серце. Це призвело до збільшення сили серцевих скорочень, тому що через мембрану типових кардіоміоцитів збільшився: In the experiment, the branches of the sympathetic nerve, which innervate the heart, are irritated. This led to an increase in the force of heart contractions, because through the membrane of typical cardiomyocytes increased:

Вхід іонів калію Input of potassium ions

Вихід іонів кальцію Output of calcium ions

Вхід іонів кальцію Input of calcium ions

Вхід іонів кальцію та калію Input of calcium and potassium ions

Вихід іонів калію Output of potassium ions

112 / 200
У пацієнта 60-ти років виявлено збільшення порогу сприймання звуків високої частоти. Зміна функцій яких структур слухового аналізатора зумовлює виникнення цього порушення? In a 60-year-old patient, an increase in the threshold for the perception of high-frequency sounds was detected. A change in the functions of which auditory analyzer structures causes this disturbance?

М’язів середнього вуха Middle ear muscles

Барабанної перетинки Eardrum

Органу Корті ближче до овального віконця The organ of Corti is closer to the oval window

Органу Корті ближче до гелікотреми The organ of Corti is closer to the helicotrema

Євстахієвої труби Eustachian tube

113 / 200
У 3-річної дитини тривале підвищення температури, збільшені лімфовузли, у крові - значне підвищення кількості лімфоцитів. Методом ІФА виявлено антиген віруса Епштейна-Бара. Який діагноз можна поставити на основі вказаного? A 3-year-old child has a long-term fever, enlarged lymph nodes, a significant increase in the number of lymphocytes in the blood. Epstein-Barr virus antigen was detected by ELISA. What diagnosis can be made on the basis of the specified?

Лімфома Беркета Burkett's lymphoma

Іенералізована інфекція, викликана herpes-zoster Generalized infection caused by herpes-zoster

Цитомегаловірусна інфекція Cytomegalovirus infection

Іерпетична аденопатія Herpetic adenopathy

Інфекційний мононуклеоз Infectious mononucleosis

114 / 200
У дитини виявлена схильність до ожиріння, яка є результатом діатезу. Назвіть вид діатезу, при якому частіше може розвинутись ожиріння: The child has a tendency to obesity, which is the result of a diathesis. Name the type of diathesis in which obesity can develop more often:

Астенічний Asthenic

Лімфатико-гіпопластичний Lymphatic-hypoplastic

- -

Нервово-артритичний Nervous-arthritic

Ексудативно-катаральний Exudative-catarrhal

115 / 200
У юнака 18-ти років діагностовано хворобу Марфана. При дослідженні встановлено порушення розвитку сполучної тканини, будови кришталика ока, аномалії серцево-судинної системи, арахнодактилію. Прикладом якого генетичного явища є ця хвороба? An 18-year-old young man was diagnosed with Marfan's disease. During the examination, a violation of the development of connective tissue, the structure of the lens of the eye, abnormalities of the cardiovascular system, and arachnodactyly were found. An example of which genetic phenomenon is this disease?

Плейотропія Pleiotropy

Комплементарність Complementarity

Множинний алелізм Multiple allelism

Неповне домінування Incomplete dominance

Кодомінування Codominance

116 / 200
Ураження хворого одноразовою дозою іонізуючого випромінювання спричинило розвиток кістково-мозкової форми променевої хвороби. Які патологічні прояви з боку крові будуть характерними в період удаваного благополуччя? Injury of the patient with a single dose of ionizing radiation caused the development of the bone-cerebral form of radiation sickness. What pathological manifestations on the part of the blood will be characteristic during the period of feigned well-being?

Анемія, лейкопенія Anemia, leukopenia

Перерозподільчий лейкоцитоз, лім-фоцитоз Redistributive leukocytosis, lymphocytosis

Наростаюча лімфопенія, лейкопенія Increasing lymphopenia, leukopenia

Тромбоцитопенія, лейкоцитоз Thrombocytopenia, leukocytosis

Тромбоцитопенія, анемія Thrombocytopenia, anemia

117 / 200
У хворого на черевний тиф при проведенні серологічного дослідження (реакція Відаля) виявлено О- і Н-аглютитиніни у титрі 1:800 і 1:200 відповідно. Це свідчить про: O- and H-agglutitinins were detected in the titer of 1:800 and 1:200, respectively, in a patient with typhoid fever during a serological examination (Vidal's reaction). This indicates about:

Неможливість підтвердити діагноз Unable to confirm diagnosis

Період реконвалесценції Period of convalescence

Проведене щеплення Vaccination carried out

Початок захворювання Start of disease

Раніше перенесене захворювання Previous illness

118 / 200
У дитячому колективі проведено планову вакцинацію проти кору. Яким методом можна перевірити ефективність проведеної вакцинації? A scheduled vaccination against measles was carried out in the children's team. What method can be used to check the effectiveness of the vaccination?

Вірусологічний Virus

Алергопроба Allergy test

Біологічний Biological

Серологічний Serological

Вірусоскопічний Virusoscopic

119 / 200
У клінічній практиці застосовують для лікування туберкульозу препарат ізоніазид - антивітамін, який здатний проникати у туберкульозну палочку. Туберкулостатичний ефект обумовлений порушенням процесів реплікації, окисно-відновних реакцій завдяки утворенню несправжнього коферменту з: In clinical practice, the drug isoniazid is used for the treatment of tuberculosis - an antivitamin that is able to penetrate into the tubercle bacillus. The tuberculostatic effect is due to the violation of replication processes, redox reactions due to the formation of a false coenzyme with:

КоО KoO

ФАД FAD

ФМН FMN

ТДФ TDF

НАД OVER

120 / 200
При огляді хворої лікар-гінеколог відмітив симптоми запалення статевих шляхів, у мазку взятому із піхви, виявлено грушоподібні найпростіші з шипом, з передньої частини відходять джгутики, наявна ундулююча мембрана. Яке захворювання підозрює лікар у хворої? During the examination of the patient, the gynecologist noticed symptoms of inflammation of the genital tract, in a smear taken from the vagina, pear-shaped protozoa with a thorn were found, flagella come from the front part, an undulating membrane is present What disease does the doctor suspect in the patient?

Кишковий трихомоноз Intestinal trichomoniasis

Токсоплазмоз Toxoplasmosis

Урогенітальний трихомоноз Urogenital trichomoniasis

Балантидіоз Balantidiosis

Лямбліоз Giardiasis

121 / 200
У чоловіка 32-х років, хворого на пневмонію, спостерігається закупорка харкотинням дихальних шляхів. В організмі хворого при цьому буде розвиватися така зміна кислотно-лужної рівноваги: A 32-year-old man with pneumonia has a sputum blockage of the respiratory tract. At the same time, the patient's body will develop the following change in acid-alkaline balance:

Метаболічний ацидоз Metabolic acidosis

Респіраторний алкалоз Respiratory alkalosis

Метаболічний алкалоз Metabolic alkalosis

Змін не буде There will be no changes

Респіраторний ацидоз Respiratory acidosis

122 / 200
При аналiзi ЕКГ виявлено випадіння деяких серцевих циклів PQRST. Наявні зубці та комплекси не змінені. Назвать вид аритмії: During the ECG analysis, the loss of some PQRST cardiac cycles was detected. The existing teeth and complexes are not changed. Name the type of arrhythmia:

Атріовентрикулярна блокада Atrioventricular block

Синоатріальна блокада Sinoatrial block

Внутрішньопередсердна блокада Intra-atrial blockade

Миготлива аритмія Atrial fibrillation

Передсердна екстрасистола Atrial extrasystole

123 / 200
У хворої встановлено порушення виділення тиреотропного гормону гіпофіза. Зі зниженням функцій якої частки гіпофіза це пов’язано? The patient has been diagnosed with a violation of the release of thyroid-stimulating hormone from the pituitary gland. This is due to a decrease in the functions of which part of the pituitary gland?

Infundibulum Infundibulum

Pars intermedia Pars intermedia

Lobus anterior Lobus anterior

- -

Lobus posterior Lobus posterior

124 / 200
Стресовий стан і больове відчуття у пацієнта перед візитом до стоматолога супроводжуються анурією (відсутністю сечовиділення). Це явище зумовлене збільшенням: The patient's stress and pain before visiting the dentist are accompanied by anuria (absence of urination). This phenomenon is caused by an increase in:

Секреції вазопресину та зменшенням адреналіну Secretion of vasopressin and reduction of adrenaline

Секреції вазопресину та адреналіну Secretions of vasopressin and epinephrine

Секреції адреналіну та зменшенням вазопресину Secretions of epinephrine and reduction of vasopressin

Активності антиноціцептивної системи Activities of the antinociceptive system

Активності парасимпатичної нервової системи Activities of the parasympathetic nervous system

125 / 200
Введення знеболюючого пацієнту перед екстракцією зуба призвело до розвитку анафілактичного шоку, який супроводжувався розвитком олігурії. Який патогенетичний механізм зумовив зменшення діурезу в даній клінічній ситуації? The administration of an anesthetic to a patient before tooth extraction led to the development of anaphylactic shock, which was accompanied by the development of oliguria. What pathogenetic mechanism caused the decrease in diuresis in this clinical situation?

Збільшення онкотичного тиску крові Increased blood oncotic pressure

Зниження гідростатичного тиску в капілярах клубочків Reduction of hydrostatic pressure in glomerular capillaries

Пошкодження клубочкового фільтру Damage of the glomerular filter

Підвищення гідростатичного тиску в капсулі Шумлянського-Боумена Increase in hydrostatic pressure in the Shumlyansky-Bowman capsule

Зменшення кількості функціонуючих нефронів Decreasing the number of functioning nephrons

126 / 200
Хвора на ревматоїдний артрит після трьохтижневого лікування преднізолоном почала скаржитись на перебої в роботі серця. З чим пов’язаний розвиток даного небажаного ефекту препарату? After a three-week treatment with prednisolone, a patient with rheumatoid arthritis began to complain of interruptions in the work of the heart. What is the development of this undesirable effect of the drug?

Гіпоглікемія Hypoglycemia

Гіперурікемія Hyperuricemia

Гіпокаліємія Hypokalemia

Гіперглікемія Hyperglycemia

Гіперкаліємія Hyperkalemia

127 / 200
У хворого 45-ти років на тлі трансмурального інфаркту міокарда розвинулася гостра лівошлуночкова недостатність. Який лікарський засіб доцільно застосувати у даній ситуації для покращення помпової функції серця? A 45-year-old patient developed acute left ventricular failure against the background of a transmural myocardial infarction. What medicine should be used in this situation to improve the pumping function of the heart?

Ізадрин Izadrin

Добутамін Dobutamine

Ефедрин Ephedrine

Промедол Promedol

Еуфілін Euphilin

128 / 200
Для профілактики атеросклерозу, ішемічної хвороби серця, порушень мозкового кровообігу рекомендується споживання жирів із високим вмістом поліненасичених жирних кислот. Однією з таких жирних кислот є: For the prevention of atherosclerosis, coronary heart disease, disorders of cerebral circulation, it is recommended to consume fats with a high content of polyunsaturated fatty acids. One of these fatty acids is:

Стеаринова Stearinova

Лауринова Laurinova

Лінолева Linoleum

Пальмітоолеїнова Palmitoolein

Олеїнова Oleinova

129 / 200
У хворого з нагноєнням рани при бактеріологічному дослідженні ранового вмісту виявлено грамнегативну паличку, яка на МПА утворює напівпрозорі слизові колонії синьо-зеленого кольору з перламутровим відтінком. Культура має специфічний запах фіалок або жасмину. Який вид збудника виділений з рани хворого? In a patient with suppuration of a wound, a gram-negative bacillus was found during bacteriological examination of the wound contents, which on MPA forms translucent mucous colonies of blue-green color with a mother-of-pearl shade. The culture has a specific smell violets or jasmine. What type of pathogen was isolated from the patient's wound?

S. pyogenes S. pyogenes

S. faecalis S. faecalis

P vulgaris P vulgaris

P aeruginosa P aeruginosa

S. aureus S. aureus

130 / 200
Після обстеження пацієнта в клініці нервових хвороб встановлена відсутність звуження зіниці при дії світла. З ураженням яких структур головного мозку це пов’язано? After examining the patient in the clinic of nervous diseases, it was established that there is no constriction of the pupil when exposed to light. What brain structures is this affected?

Ядра гіпоталамуса Nuclei of the hypothalamus

Ретикулярш ядра довгастого мозку Reticularis of the nucleus of the medulla oblongata

Червоні ядра середнього мозку Red nuclei of the midbrain

Вегетативні ядра 3 пари черепно-мозкових нервів Vegetative nuclei of 3 pairs of cranial nerves

Ретикулярні ядра середнього мозку Reticular nuclei of the midbrain

131 / 200
У хворого під час комп’ютерної томографії грудної клітки діагностовано пухлину заднього нижнього середостіння. Яка з перерахованих структур стиснута пухлиною? The patient was diagnosed with a tumor of the posterior lower mediastinum during a chest CT scan. Which of the listed structures is compressed by the tumor?

Arcus aortae Arcus aortae

Trachea Trachea

Vena cava superior Vena cava superior

Aorta thoracica Aorta thoracica

N. phrenicus N. phrenicus

132 / 200
Під час операції холецистектомії у хірурга виникла необхідність визначити топографію загальної жовчної протоки. Злиттям яких проток утворюється дана анатомічна структура? During the cholecystectomy operation, the surgeon needed to determine the topography of the common bile duct. The fusion of which ducts forms this anatomical structure?

Правої та лівої печінкових проток Right and left hepatic ducts

Загальної печінкової і правої печінкової проток Common hepatic and right hepatic duct

Лівої печінкової і міхурової проток Left hepatic and cystic duct

Загальної печінкової і лівої печінкової проток Common hepatic and left hepatic duct

Загальної печінкової і міхурової проток Common hepatic and cystic duct

133 / 200
Дитина 6-ти років під час гри порізала ногу осколком скла і була направлена у поліклініку для введення протиправцевої сироватки. З метою попередження розвитку анафілактичного шоку лікувальну сироватку вводили за методом Безредка. Який механізм лежить в основі подібного способу гіпосенсибілізації організму? A 6-year-old child cut his leg with a shard of glass while playing and was sent to the polyclinic for administration of anti-tetanus serum. In order to prevent the development of anaphylactic shock, the therapeutic serum was administered using the method Bezredka. What mechanism underlies this method of hyposensitization of the body?

Зв’язування фіксованих на тучних клітинах IgE Mat cell-fixed IgE binding

Блокування синтезу медіаторів у тучних клітинах Blocking of synthesis of mediators in mast cells

Стимуляція імунологічної толерантності до антигену Stimulation of immunological tolerance to antigen

Зв’язування рецепторів до IgE на тучних клітинах IgE receptor binding on mast cells

Стимуляція синтезу антиген-специфічних IgG Stimulation of synthesis of antigen-specific IgG

134 / 200
Хворому лікар призначив протикашльовий препарат центральної дії, який є алкалоїдом мачку жовтого. Діє на кашльовий центр вибірково, не пригнічуючи дихання, не затримує виділення харкотиння. Не викликає обстипації і лікарської залежності. Можна призначати дітям. Визначте препарат: The doctor prescribed an antitussive drug of central action to the patient, which is an alkaloid of yellow maca. It acts on the cough center selectively, without suppressing breathing, and does not delay the release of sputum. It does not cause constipation and drug addiction. Can be prescribed to children. Identify the drug:

Бромгексин Bromhexine

Кодеїну фосфат Codeine Phosphate

Лібексин Libexin

Окселадин Oxeladine

Глауцину гідрохлорид Glaucine hydrochloride

135 / 200
Хлопчику 5-ти років був встановлений діагноз - міастенія. Оберіть препарат з групи антихолінестеразних засобів, який покращує нервово-м’язову передачу: A 5-year-old boy was diagnosed with myasthenia gravis. Choose a drug from the group of anticholinesterase drugs that improves neuromuscular transmission:

Алоксим Aloxim

Іалантаміну гідробромід Yalanthamine hydrobromide

Армін Armin

Прозерин Proserin

Ацеклідин Aceclidine

136 / 200
Хворому 63-х років з атонією сечового міхура лікар призначив препарат, дозу якого хворий самостійно збільшив. З’явились підвищене потовиділення, салівація, діарея, м’язові спазми. Препарат якої групи був призначений? The doctor prescribed a drug to a 63-year-old patient with atony of the bladder, the dose of which the patient increased on his own. Increased sweating, salivation, diarrhea, muscle spasms appeared What group of drugs was prescribed?

Токолітики Tocolytics

Адреноблокатори Adrenoblockers

Реактиватори холінестерази Cholinesterase reactivators

Гангліоблокатори Ganglioblockers

Холіноміметики Cholinomimetics

137 / 200
Під час розтину тіла жінки 28-ми років, яка померла від геморагічного шоку, виявлено: гемоперитонеум, права маткова труба збільшена у розмірах, багряна, з наскрізним дефектом стінки, її дилятований просвіт виповнений темно-червоними згортками крові. Гістологічно у слизовій оболонці труби та серед м’язових клітин визначаються пласти великих світлих децидуальних клітин, у м’язовій оболонці та серед згортків крові у просвіті труби - ворсинки хоріона. Діагностуйте патологію вагітності: During the autopsy of the body of a 28-year-old woman who died of hemorrhagic shock, it was found: hemoperitoneum, the right fallopian tube was enlarged, purple, with a through wall defect , its dilated lumen is filled with dark red blood clots. Histologically, in the mucous membrane of the tube and among the muscle cells, layers of large light decidual cells are determined, in the muscle membrane and among the blood clots in the lumen of the tube - chorionic villi. Diagnose the pathology of pregnancy:

Порушена маткова вагітність Disturbed uterine pregnancy

Деструюючий міхурцевий занесок Destructive Bubble Drift

Черевна вагітність Abdominal pregnancy

Інтерлігаментарна вагітність Interligamentous pregnancy

Порушена трубна вагітність Disrupted tubal pregnancy

138 / 200
У біоптаті нирки 45-річного чоловіка, що має хронічну хворобу нирок, виявлено: склероз, лімфо-плазмоцитарна інфільтрація стінок мисок та чашок, дистрофія та атрофія канальців. Збережені канальці розширені, розтягнені колоїдоподібними масами, епітелій сплющений (’’щитоподібна” нирка). Який діагноз найбільш імовірний? A kidney biopsy of a 45-year-old man with chronic kidney disease revealed: sclerosis, lympho-plasmacytic infiltration of the walls of bowls and cups, dystrophy and atrophy of tubules. Saved the tubules are dilated, stretched by colloid-like masses, the epithelium is flattened ('thyroid' kidney). What is the most likely diagnosis?

Тубуло-інтерстиційний нефрит Tubulo-interstitial nephritis

Гострий пієлонефрит Acute pyelonephritis

Гломерулонефрит Glomerulonephritis

Нефросклероз Nephrosclerosis

Хронічний пієлонефрит Chronic pyelonephritis

139 / 200
Чоловік 40 років хворіє на гіперацидний гастрит з нічними голодними болями. Призначте хворому лікарський засіб - блокатор гістамінових H2-рецепторів III покоління, який знизить виділення хлористоводневої кислоти (особливо вночі) та збільшить утворення захисного слизу: A 40-year-old man suffers from hyperacid gastritis with nocturnal hunger pangs. Prescribe the patient a medication - a histamine H2-receptor blocker of the III generation, which will reduce the release of hydrochloric acid (especially at night ) and will increase the formation of protective mucus:

Фамотидин Famotidine

Атропіну сульфат Atropine sulfate

Платифіліну гідротартрат Platiphylline hydrotartrate

Пірензепін Pirenzepine

Метацин Metacin

140 / 200
Під час футбольного матчу між вболівальниками різних команд виникла сутичка. На фоні негативних емоцій в одного учасника сутички були розширені зіниці й підвищене серцебиття. Активація якої системи регуляції функцій організму забезпечує такі вегетативні зміни при негативних емоціях? During a football match, a fight broke out between fans of different teams. Against the background of negative emotions, one participant in the fight had dilated pupils and an increased heart rate. Activation of which system of regulation of body functions provides such autonomic changes during negative emotions?

Гіпоталамо-гіпофізарно-тиреоїдна Hypothalamus-pituitary-thyroid

Метасимпатична нервова Metasympathetic nerve

Симпато-адреналова Sympathoadrenal

Парасимпатична нервова Parasympathetic nerve

Соматична нервова Somatic nerve

141 / 200
У хворого 32-х років після оперативного втручання розвинувся гнійний процес. Із гною рани виділено культуру S. aureus. Який з перерахованих тестів найдоцільніше використати для диференціації S. aureus від S. epidermidis? A 32-year-old patient developed a purulent process after surgery. A culture of S. aureus was isolated from the pus of the wound. Which of the listed tests is most appropriate to use for differentiation of S. aureus from S. epidermidis?

Колір колонії Colony Color

Плазмокоагулазна активність Plasmocoagulase activity

Ферментація арабінози Arabinose fermentation

Оксидазний тест Oxidase test

Гемоліз на кров’яному агарі Hemolysis on blood agar

142 / 200
Чоловікові 58-ми років зроблено операцію з приводу раку простати. Через 3 місяці йому проведено курс променевої та хіміотерапії. До комплексу лікарських препаратів входив 5-фтордезоксиуридин - інгібітор тимідилатсинтази. Синтез якої речовини блокується цим препаратом? A 58-year-old man was operated on for prostate cancer. After 3 months, he underwent a course of radiation and chemotherapy. The complex of drugs included 5-fluorodeoxyuridine - a thymidylate synthase inhibitor . The synthesis of which substance is blocked by this drug?

ДНК DNA

- -

т-РНК t-RNA

р-РНК p-RNA

і-РНК i-RNA

143 / 200
В підводному човні під час занурення порушилася система подачі кисню. У підводників збільшилися частота дихання і серцевих скорочень. Який вид гіпоксії розвинувся у підводників? The oxygen supply system was broken in the submarine during the dive. The breathing and heart rate of the submariners increased. What type of hypoxia developed in the submariners?

Гіпоксична Hypoxic

Дихальна Respiratory

Серцево-судинна Cardiovascular

Тканинна Fabric

Кров’яна Bloody

144 / 200
Синтез і-РНК проходить на матриці ДНК з урахуванням принципу комплементарності. Якщо триплети у ДНК наступні - АТГ-ЦГТ, то відповідні кодони і-РНК будуть: The synthesis of i-RNA takes place on the DNA matrix taking into account the principle of complementarity. If the triplets in DNA are as follows - ATG-CHT, then the corresponding codons of i-RNA will be:

ТАГ-УГУ TAG-UGU

УАЦ-ГЦА UAC-HCA

АТГ-ЦГТ ATG-CHT

АУГ-ЦГУ AUG-CSU

УАГ-ЦГУ UAG-CSU

145 / 200
У процесі фібринолізу кров’яний тромб розсмоктується. Розщеплення нерозчинного фібрину відбувається шляхом його гідролізу під дією протеолітичного ферменту плазміну, який наявний у крові в неактивній формі плазміногену. Активується плазміноген шляхом обмеженого протеолізу за участю фермента: In the process of fibrinolysis, the blood clot dissolves. Insoluble fibrin is broken down by its hydrolysis under the action of the proteolytic enzyme plasmin, which is present in the blood in the inactive form of plasminogen. Plasminogen is activated by limited proteolysis with the participation of an enzyme:

Урокіназа Urokinase

Трипсин Trypsin

Пепсин Pepsin

Ентерокіназа Enterokinase

Хімотрипсин Chymotrypsin

146 / 200
У біоптаті щитоподібної залози виявлено атрофію паренхіматозних елементів, дифузну інфільтрацію тканини залози лімфоцитами та плазматичними клітинами з утворенням в ній лімфоїдних фолікулів. Для якого захворювання є характерними наведені ознаки? Atrophy of parenchymal elements, diffuse infiltration of the gland tissue by lymphocytes and plasma cells with the formation of lymphoid follicles were found in the biopsy of the thyroid gland. For which disease are these symptoms characteristic?

Хвороба Базедова Bazedov's disease

Аденома щитоподібної залози Thyroid adenoma

Ендемічний зоб Endemic goiter

Тиреоїдит Хасімото Hashimoto's thyroiditis

Тиреоїдит Ріделя Riedel's thyroiditis

147 / 200
У хворого з жовтяницею встановлено: підвищення у плазмі крові вмісту загального білірубіну за рахунок непрямого (вільного), в калі і сечі - високий вміст стеркобіліну, рівень прямого (зв’язаного) білірубіну у плазмі крові в межах норми. Який вид жовтяниці має місце у хворого? In a patient with jaundice, it was established: an increase in the content of total bilirubin in the blood plasma due to indirect (free), in feces and urine - a high content of stercobilin, the level of direct (with bilirubin in the blood plasma is within the normal range. What type of jaundice does the patient have?

Механічна Mechanical

- -

Гемолітична Hemolytic

Паренхіматозна Parenchymatous

Хвороба Жильбера Gilbert's disease

148 / 200
При гістологічному дослідженні біоптатів, взятих з потовщених країв виразки шлунка, виявлені невеликі гніздові скупчення різко атипових гіперхромних невеликих епітеліальних клітин, які розташовані серед дуже розвиненої строми. Визначте пухлину: On histological examination of biopsies taken from the thickened edges of a gastric ulcer, small nested clusters of sharply atypical hyperchromic small epithelial cells were found, which are located among a highly developed stroma. Identify the tumor:

Аденокарцинома Adenocarcinoma

Скіррозний недиференційований рак Scarcerous undifferentiated cancer

Недиференційована саркома Undifferentiated sarcoma

Аденома Adenoma

Медулярний рак Medullary cancer

149 / 200
Жінка 69-ти років довго хворіла на атеросклероз. Поступила в хірургічне відділення з симптомами гострого живота. При лапаротомії виявлені: тромбоз мезентеріальної артерії, петлі тонкої кишки набряклі, багряно-чорного кольору, на їх серозній оболонці фібринозні нашарування. Який патологічний процес розвинувся у кишці хворої? A 69-year-old woman suffered from atherosclerosis for a long time. She was admitted to the surgical department with acute abdominal symptoms. At laparotomy, the following were found: thrombosis of the mesenteric artery, the loops of the small intestine were swollen, purple - black in color, fibrinous layering on their serous membrane. What pathological process developed in the patient's intestine?

Суха гангрена Dry gangrene

Коагуляційний некроз Coagulation necrosis

Немічний інфаркт Nemic heart attack

Секвестр Sequestration

Волога гангрена Wet gangrene

150 / 200
Під час ректороманоскопії хворого зі скаргами на діарею виявлено, що слизова оболонка прямої і сигмоподібної кишок різко гіперемована, набрякла, вкрита великою кількістю слизу, а у деяких ділянках вкрита плівчастими накладаннями зеленуватого кольору. Про яке захворювання можна думати? During the rectoromanoscopy of a patient with complaints of diarrhea, it was found that the mucous membrane of the rectum and sigmoid intestine is sharply hyperemic, swollen, covered with a large amount of mucus, and in some areas covered with membranous greenish overlays. What disease can you think of?

Черевний тиф Typhoid

Холера Cholera

Дизентерія Dysentery

Амебіаз Amebiasis

Сальмонельоз Salmonellosis

151 / 200
При копрологічному дослідженні у працівників кав’ярні лікарями санітарно- епідеміологічної станції були виявлені округлі цисти, характерною ознакою яких є наявність чотирьох ядер. Імовірніше за все у цих працівників безсимптомно паразитує: During the coprological examination of coffee shop workers, the doctors of the sanitary and epidemiological station found rounded cysts, the characteristic feature of which is the presence of four nuclei. Most likely, these workers are asymptomatic parasitizes:

Балантидій Balantidius

Кишкова трихомонада Intestinal trichomonas

Дизентерійна амеба Dysenteric amoeba

Амеба кишкова Intestinal amoeba

Лямблія Lamblia

152 / 200
У хворого з варикозним розширенням вен під час огляду нижніх кінцівок відзначається: ціаноз, пастозність, зниження температури шкіри, поодинокі петехії. Який розлад гемодинаміки має місце у хворого? In a patient with varicose veins, during the examination of the lower extremities, the following is noted: cyanosis, pastiness, a decrease in skin temperature, isolated petechiae. What hemodynamic disorder does the patient have?

Компресійна ішемія Compression ischemia

Тромбоемболія Thromboembolism

Обтураційна ішемія Obstructive ischemia

Артеріальна гіперемія Arterial hyperemia

Венозна гіперемія Venous hyperemia

153 / 200
При гістологічному дослідженні органів і тканин померлої від ниркової недостатності молодої жінки, у якої прижиттєво виявлявся високий титр антинуклеарних антитіл, виявлені поширені фібриноїдні зміни в стінках судин. Відмічається ядерна патологія з вакуолізацією ядер, каріорексисом, утворенням гематоксилінових тілець. Який найбільш імовірний діагноз? During a histological examination of the organs and tissues of a young woman who died of kidney failure, in whom a high titer of antinuclear antibodies was detected during life, widespread fibrinoid changes in the vessel walls were detected. Nuclear pathology is noted with vacuolization of nuclei, karyorrhexis, formation of hematoxylin bodies. What is the most likely diagnosis?

Облітеруючий ендартеріїт Endarteritis obliterans

Системний червоний вовчак Systemic lupus erythematosus

Вузликовий періартеріїт Nodular periarteritis

Гіпертонічна хвороба Hypertensive disease

Атеросклероз Atherosclerosis

154 / 200
Мікоплазми є своєрідною групою мікроорганізмів, що відносяться до родини Mycoplasmataceae і що мають властивості як бактерій, так і вірусів. Назвіть одну особливість мікоплазм, яка відрізняє їх від бактерій і вірусів: Mycoplasmas are a peculiar group of microorganisms that belong to the family Mycoplasmataceae and have properties of both bacteria and viruses. Name one feature of mycoplasmas that distinguishes them from bacteria and viruses:

Відсутність клітинної будови No cellular structure

Спосіб розмноження Reproduction method

Внутрішньоклітинний паразитизм Intracellular parasitism

Відсутність клітинної стінки No cell wall

Висока ферментативна активність High enzymatic activity

155 / 200
Після довготривалого вживання антибіотиків у хворого на слизовій ротової порожнини появилися округлі білі плями, на язику білий наліт. Який мікроорганізм імовірно спричинив дані симптоми? After long-term use of antibiotics, the patient developed round white spots on the mucous membrane of the oral cavity, a white plaque on the tongue. What microorganism probably caused these symptoms?

Кишкова паличка Escherichia coli

Стрептокок Streptococcus

Гриби роду Candida Candida fungi

Ентерокок Enterococcus

Лактобацили Lactobacilli

156 / 200
Фенілкетонурія - це захворювання, яке зумовлено рецесивним геном, що локалізується в аутосомі. Батьки є гетерозиготами за цим геном. Вони вже мають двох хворих синів і одну здорову дочку. Яка імовірність, що четверта дитина, яку вони очікують народиться теж хворою? Phenylketonuria is a disease caused by an autosomal recessive gene. The parents are heterozygous for this gene. They already have two sick sons and one healthy daughter. What is the probability that the fourth child they expect will also be sick?

0% 0%

25% 25%

100% 100%

50% 50%

75% 75%

157 / 200
При диспансерному обстеженні у хворого знайдено цукор в сечі. Який найбільш імовірний механізм виявлених змін, якщо вміст цукру в крові нормальний? During the dispensary examination, sugar was found in the patient's urine. What is the most probable mechanism of the detected changes, if the blood sugar content is normal?

Недостатня продукція інсуліну підшлунковою залозою Insufficient production of insulin by the pancreas

Порушення фільтрації глюкози в клубочковому відділі нефрона Disturbance of glucose filtration in the glomerular part of the nephron

!нсулінорезистентність рецепторів клітин !insulin resistance of cell receptors

Гіперпродукція глюкокортикоїдів наднирниками Hyperproduction of glucocorticoids by the adrenal glands

Порушення реабсорбції глюкози в канальцях нефрона Disturbance of glucose reabsorption in nephron tubules

158 / 200
В пробірку, що містить розчин NaCl 0,9%, додана крапля крові. Що відбудеться з еритроцитами? A drop of blood was added to a test tube containing a 0.9% NaCl solution. What will happen to the erythrocytes?

Залишаться без змін Remain unchanged

Зморшкування Wrinkle

Набухання Swelling

Осмотичний гемоліз Osmotic hemolysis

Біологічний гемоліз Biological hemolysis

159 / 200
У хворого 40-ка років ознаки гірської хвороби: запаморочення, задишка, тахікардія, рН крові - 7,50, pCO2 - 30 мм рт.ст., зсув буферних основ +4 ммоль/л. Яке порушення кислотно-основного стану має місце? A 40-year-old patient has signs of mountain sickness: dizziness, shortness of breath, tachycardia, blood pH - 7.50, pCO2 - 30 mm Hg, shift buffer bases +4 mmol/l. What violation of the acid-base state is taking place?

Негазовий ацидоз Nongaseous acidosis

Газовий алкалоз Gas alkalosis

Газовий ацидоз Gas acidosis

Негазовий алкалоз Nongaseous alkalosis

Видільний ацидоз Excretory acidosis

160 / 200
У хворого 15-ти років концентрація глюкози натще - 4,8 ммоль/л, через годину після цукрового навантаження - 9,0 ммоль/л, через 2 години - 7,0 ммоль/л, через 3 години - 4,8 ммоль/л. Ці показники характерні для такого захворювання: A 15-year-old patient has a fasting glucose concentration of 4.8 mmol/l, an hour after a sugar load - 9.0 mmol/l, after 2 hours - 7.0 mmol/l, after 3 hours - 4.8 mmol/l. These indicators are typical for such a disease:

Цукровий діабет I типу Type I diabetes

Прихований цукровий діабет Hidden diabetes

Хвороба !ценко-Кутінга. Tsenko-Kuting disease.

Цукровий діабет II типу Type II diabetes

- -

161 / 200
Лікар призначив хворому з гострою серцевою недостатністю не-глікозидний кардіотонічний засіб, який безпосередньо стимулює в1-адренорецептори міокарда, що збільшує кровообіг, діурез. Застосовується лише внутрішньовенно крапельно внаслідок швидкої інактивації в організмі. Який препарат призначив лікар? The doctor prescribed a non-glycoside cardiotonic agent to a patient with acute heart failure, which directly stimulates β1-adrenoceptors of the myocardium, which increases blood circulation, diuresis. It is used only intravenously as a result of rapid inactivation in the body. What drug did the doctor prescribe?

Добутамін Dobutamine

Дигоксин Digoxin

Анаприлін Anaprilin

Корглікон Corglycon

Адреналін Adrenaline

162 / 200
Чоловік 40-ка років перебував у пульмонологічному відділенні з приводу рецидивуючої правосторонньої пневмонії. Помер від легенево-серцевої недостатності. На розтині в правій легені визначається ділянка круглої форми 3х4 см. Вона являє собою порожнину з нерівними шорсткими краями, заповнену каламутною вершкоподібною жовто-зеленою рідиною. Мікроскопічно: стінка порожнини утворена тканиною легені з дифузною інфільтрацією лейкоцитами. Визначте патологічний процес у легені: A 40-year-old man was in the pulmonology department due to recurrent right-sided pneumonia. He died of pulmonary heart failure. An autopsy revealed a round 3x4 cm area in the right lung . It is a cavity with uneven rough edges, filled with a cloudy creamy yellow-green liquid. Microscopically: the wall of the cavity is formed by lung tissue with diffuse infiltration of leukocytes. Define the pathological process in the lung:

Хронічний абсцес Chronic abscess

Гангрена Gangrene

Гострий абсцес Acute abscess

Емпієма Empyema

!нфаркт !heart attack

163 / 200
У вагітної жінки 24-х років після тривалого блювання було зареєстровано зниження об’єму циркулюючої крові. Про яку зміну загальної кількості крові може йти мова? A 24-year-old pregnant woman had a decrease in the volume of circulating blood after prolonged vomiting. What kind of change in the total amount of blood can we be talking about?

Олігоцитемічна гіперволемія Oligocythemic hypervolemia

Поліцитемічна гіперволемія Polycythemic hypervolemia

Олігоцитемічна гіповолемія Oligocythemic hypovolemia

Проста гіповолемія Simple hypovolemia

Поліцитемічна гіповолемія Polycythemic hypovolemia

164 / 200
Пацієнту, який знаходився в клініці з приводу пневмонії, ускладненої плевритом, у складі комплексної терапії вводили преднізолон. Протизапальна дія цього синтетичного глюкокортикоїда пов’язана з блокуванням вивільнення арахідонової кислоти шляхом гальмування: A patient who was in the clinic for pneumonia complicated by pleurisy was administered prednisolone as part of complex therapy. The anti-inflammatory effect of this synthetic glucocorticoid is associated with blocking the release of arachidonic acid by braking:

Пероксидази Peroxidases

Фосфоліпази С Phospholipase C

Фосфоліпази A2 Phospholipase A2

Циклооксигенази Cyclooxygenases

Ліпоксигенази Lipoxygenases

165 / 200
У чоловіка 64-х років, який тривалий час курив і вживав міцні спиртні напої, на боковій поверхні язика виявили подібний на виразку утвір із білої, помірно щільної тканини розмірами 5х3 см. При гістологічному дослідженні біоптату виявили, що утвір побудований із клітини, які формують солідні структури і тяжі, що нагадують за будовою багатошаровий плоский епітелій, в якому клітини з вираженим поліморфізмом, з великими атиповими ядрами з патологічними мітозами. Діагностуйте виявлене у чоловіка захворювання: In a 64-year-old man who smoked and drank strong alcoholic drinks for a long time, an ulcer-like formation of white, moderately dense tissue with the size of 5x3 cm. During the histological examination of the biopsy, it was found that the formation is made of cells that form solid structures and rods, resembling in structure a multilayered flat epithelium, in which cells with pronounced polymorphism, with large atypical nuclei with pathological mitoses. Diagnose the disease detected in a man :

Плоскоклітинний зроговілий рак Squamous keratinized cancer

Рак на місці Cancer in place

Еритроплакія Erythroplakia

Лейкоплакія Leukoplakia

Плоскоклітинний незроговілий рак Squamous nonkeratinous cancer

166 / 200
Обстежуваний знаходиться у фазі повільнохвильового глибокого сну. Про це свідчить реєстрація на ЕЕГ таких хвиль: The subject is in the phase of slow-wave deep sleep. This is evidenced by the registration of the following waves on the EEG:

Тета -хвилі Theta Waves

Дельта-хвилі Delta Waves

Альфа-веретена Alpha Spindle

Альфа-хвилі Alpha Waves

Бета-хвилі Beta Waves

167 / 200
Лікар-стоматолог для лікування гінгівіту призначив пацієнту препарат з протипротозойною та антибактеріальною діями, який може викликати відразу до алкоголю. Вкажіть препарат, який призначив лікар: To treat gingivitis, the dentist prescribed the patient a drug with antiprotozoal and antibacterial effects, which can cause an aversion to alcohol. Specify the drug prescribed by the doctor:

Цефтріаксон Ceftriaxone

Тетрациклін Tetracycline

Левоміцетин Levomycetin

Лінкоміцину гідрохлорид Lincomycin hydrochloride

Метронідазол Metronidazole

168 / 200
Спеціальний режим харчування призвів до зменшення іонів Ca2+ в крові. До збільшення секреції якого гормону це призведе? A special diet led to a decrease in Ca2+ ions in the blood. Which hormone will this lead to an increase in secretion?

Соматотропін Somatotropin

Тироксин Thyroxine

Вазопресин Vasopressin

Паратгормон Parathyroid hormone

Тирокальцитонін Tyrocalcitonin

169 / 200
У пацієнта після переохолодження на губах з’явились герпетичні висипання. Для лікування призначений крем ацикловіру, терапевтичний ефект якого пояснюється здатністю: The patient developed herpetic rashes on his lips after hypothermia. Acyclovir cream is prescribed for treatment, the therapeutic effect of which is explained by the ability to:

Уповільнювати реплікацію РНК- і ДНК-геномних вірусів Slow down the replication of RNA and DNA genomic viruses

Гальмувати активність протеази Inhibit protease activity

Інгібувати нейрамінідазу Inhibit neuraminidase

Гальмувати активність ДНК-полімерази Inhibit DNA polymerase activity

Порушувати функцію зворотної транскриптази Disrupt reverse transcriptase function

170 / 200
На плановий прийом до педіатра батьки привели дитину віком 13 місяців. Під час повного огляду лікар перевірив розвиток II сигнальної системи дитини. Назвіть період, коли у людини вперше з’являються ознаки розвитку II сигнальної системи: Parents brought a 13-month-old child to a pediatrician for a routine appointment. During a complete examination, the doctor checked the development of the child's II signal system. Name the period when a person first had there are signs of the development of the II signal system:

1,5-2 роки 1.5-2 years

2,5-3 роки 2.5-3 years

6-12 місяців 6-12 months

3-5 років 3-5 years

2-2,5 роки 2-2.5 years

171 / 200
У сироватці крові новонародженого виявлено антитіла до вірусу кору. Про наявність якого імунітету це може свідчити? Antibodies to the measles virus were detected in the blood serum of a newborn. What kind of immunity does this indicate?

Штучний пасивний Artificial Passive

Природний активний Natural Active

Штучний активний Artificial Active

Природний пасивний Natural Passive

Спадковий, видовий Hereditary, species

172 / 200
У хворого спостерігається пухлина тканин орбіти позаду очного яблука. Зазначено порушення акомодації та звуження зіниці ока. Яке анатомічне утворення ушкоджено? The patient has a tumor of the orbital tissues behind the eyeball. A violation of accommodation and narrowing of the pupil of the eye is noted. What anatomical formation is damaged?

N. nasociliaris N. nasociliaris

Ganglion ciliare Ganglion ciliare

N. opticus N. opticus

N. trochlearis N. trochlearis

N. lacrimalis N. lacrimalis

173 / 200
У хворого виявлено зміну функції привушної слинної залози. Який з вузлів вегетативної нервової системи віддає післявузлові симпатичні волокна для неї? A change in the function of the parotid salivary gland was detected in the patient. Which of the nodes of the autonomic nervous system sends postnodal sympathetic fibers to it?

Ganglion cervicale superius Ganglion cervicale superius

Ganglion submandibulare Ganglion submandibulare

Ganglion pterygopalatinum Ganglion pterygopalatinum

Ganglion oticum Ganglion oticum

Ganglion cervicothoracicum Ganglion cervicothoracicum

174 / 200
Жінка 31-го року хворіє на В!Л- інфекцію на стадії СНІД. На шкірі нижніх кінцівок, слизової оболонки піднебіння з’явились рудувато-червоні плями, яскраво-червоні вузлики різних розмірів. Один з вузликів взято на гістологічне дослідження. Виявлено багато хаотично розташованих тонкостінних судин, вистелених ендотелієм, пучки веретеноподібних клітин з наявністю ге-мосидерину. Яка пухлина розвинулась у хворої? A 31-year-old woman suffers from HIV infection at the AIDS stage. Reddish-red spots appeared on the skin of the lower extremities, the mucous membrane of the palate, bright -red nodules of various sizes. One of the nodules was taken for histological examination. Many thin-walled vessels lined with endothelium, bundles of spindle-shaped cells with the presence of hemosiderin were found. What kind of tumor developed in the patient?

Гемангіома Hemangioma

Саркома Капоші Kaposi's sarcoma

Лімфома Беркіта Burkitt's lymphoma

Фібросаркома Fibrosarcoma

Лімфангіома Lymphangioma

175 / 200
У альпініста, що піднявся на висоту 5200 м, розвинувся газовий алкалоз. Що є причиною його розвитку? A mountaineer who climbed to a height of 5200 m developed gas alkalosis. What is the cause of its development?

Гіпервентиляція легенів Hyperventilation of the lungs

Гіпоксемія Hypoxemia

Гіпероксемія Hyperoxemia

Зниження температури навколишнього середовища Decreasing ambient temperature

Гіповентиляція легенів Hypoventilation of the lungs

176 / 200
У хворого хронічна нежить. Набряк слизової оболонки носової порожнини призводить до порушення функції рецепторів нюхового нерва, які розташовані в нюховій ділянці носової порожнини. Через яке утворення волокна цього нерва потрапляють в передню черепну ямку? The patient has a chronic runny nose. Swelling of the mucous membrane of the nasal cavity leads to impaired function of the receptors of the olfactory nerve, which are located in the olfactory area of the nasal cavity. Through which formation do the fibers of this nerve enter in the front cranial fossa?

Foramen ethmoidale anterior Foramen ethmoidale anterior

Lamina cribrosa os ethmoidale Lamina cribrosa os ethmoidale

Foramen incisivum Foramen incisivum

Foramen ethmoidale posterior Foramen ethmoidale posterior

Foramen sphenopalatinum Foramen sphenopalatinum

177 / 200
На гістологічному препараті в складі видовженої структури, обмеженої плазмолемою, по периферії розташовані численні ядра, а в цитоплазмі наявна поперечна посмугованість. Яка це структура? On the histological preparation, as part of an elongated structure, limited by the plasmolemma, numerous nuclei are located on the periphery, and transverse striations are present in the cytoplasm. What is this structure?

Синцитіотрофобласт Syncytiotrophoblast

Колагенове волокно Collagen fiber

Гладкий міоцит Smooth myocyte

Міосимпласт Myosymplast

Кардіоміоцит Cardiomyocyte

178 / 200
У спортсмена легкоатлета (бігуна на довгі дистанції) під час змагань розвинулась гостра серцева недостатність. В результаті чого виникла ця патологія? A track and field athlete (long-distance runner) developed acute heart failure during the competition. What caused this pathology?

Порушення вінцевого кровообігу Violation of coronary blood circulation

Прямого пошкодження міокарда Direct myocardial damage

Патологія перикарда Pericard pathology

Перевантаження серця об’ємом Heart volume overload

Перевантаження серця опором Heart overload with resistance

179 / 200
У пацієнта встановлено гіповітаміноз фолієвої кислоти, що може призвести до порушення синтезу: The patient is diagnosed with hypovitaminosis of folic acid, which can lead to impaired synthesis:

Пуринових нуклеотидів та холестерину Purine nucleotides and cholesterol

Тимідилових нуклеотидів та жирних кислот Thymidyl nucleotides and fatty acids

Гема та креатину Heme and creatine

Пуринових та тимідилових нуклеотидів Purine and thymidyl nucleotides

Цитрату та кетонових тіл Citrate and ketone bodies

180 / 200
Спадкова гіперліпопротеїнемія І типу обумовлена недостатністю ліпопротеїнліпази. Підвищення рівня яких транспортних форм ліпідів в плазмі навіть натщесерце є характерним? Hereditary hyperlipoproteinemia of type I is caused by lipoprotein lipase deficiency. An increase in the level of which transport forms of lipids in the plasma even on an empty stomach is characteristic?

Хіломікрони Chylomicrons

Модифіковані ліпопротеїни Modified lipoproteins

Ліпопротеїни дуже низької густини Very low density lipoprotein

Ліпопротеїни низької густини Low-density lipoprotein

Ліпопротеїни високої густини High-density lipoprotein

181 / 200
У людини виявлена пухлина одного з відділів головного мозку, внаслідок чого в неї порушена здатність підтримувати нормальну температуру тіла. Яка структура головного мозку пошкоджена? A person has a tumor in one of the brain departments, as a result of which his ability to maintain normal body temperature is impaired. What brain structure is damaged?

Гіпоталамус Hypothalamus

Чорна субстанція Black substance

Мозочок Cerebellum

Таламус Thalamus

Стріатум Striatum

182 / 200
До лікарні звернувся чоловік 50-ти років з розладами пам’яті, болісними відчуттями по ходу нервових стовбурів, зниженням інтелектуальних функцій, порушеннями з боку серцево-судинної системи і явищами диспепсії. В анамнезі хронічний алкоголізм. Дефіцит якого вітаміну може викликати ці симптоми? A 50-year-old man came to the hospital with memory disorders, painful sensations along the course of the nerve trunks, a decrease in intellectual functions, disorders of the cardiovascular system and phenomena of dyspepsia. There is a history of chronic alcoholism. Which vitamin deficiency can cause these symptoms?

Кальциферол Calciferol

Ніацин Niacin

Рибофлавін Riboflavin

Ретинол Retinol

Тіамін Thiamine

183 / 200
Пацієнт звернувся зі скаргами на гострий біль у правому підребер’ї. При огляді лікар звернув увагу на пожовтіння склер хворого. Лабораторно: підвищена активність АлАТ та негативна реакція на стеркобілін в калі. Для якого захворювання характерні такі симптоми? The patient complained of acute pain in the right hypochondrium. During the examination, the doctor noted the yellowing of the patient's sclera. Laboratory: increased activity of ALT and a negative reaction to stercobilin in feces. What disease is characterized by such symptoms?

Хронічний коліт Chronic Colitis

Гепатит Hepatitis

Хронічний гастродуоденіт Chronic gastroduodenitis

Гемолітична жовтяниця Hemolytic Jaundice

Хронічний гастрит Chronic gastritis

184 / 200
У хворого після тривалого психоемоційного напруження спостерігається підвищення артеріального тиску, що супроводжується серцебиттям, кардіалгіями, головним болем, запамороченням. Домінуючим у формуванні артеріальної гіпертензії у даному випадку є збільшення: After prolonged psycho-emotional stress, the patient has an increase in blood pressure, which is accompanied by palpitations, cardiac pain, headache, dizziness. The dominant factor in the formation of arterial hypertension in this case is an increase in:

Частоти серцевих скорочень Heart rate

Тонусу артеріол Arteriolar tone

Серцевого викиду Cardiac output

Тонусу венул Tonus venules

Об’єму циркулюючої крові Circulating blood volume

185 / 200
Незалежно від расової чи етнічної належності у людини розвивається комплекс морфофункціональних, біохімічних, імунологічних ознак, які обумовлюють кращу біологічну пристосованість людини до відповідного фізичного середовища. Який тип біологічної реакції представлений у людини? Regardless of race or ethnicity, a person develops a complex of morphofunctional, biochemical, and immunological features that cause better biological adaptation of a person to the corresponding physical environment. What type of biological reaction is presented in a person?

Тип зони помірного клімату Temperate climate zone type

Тропічний тип Tropical type

Арктичний тип Arctic type

Гірський тип Mountain type

Адаптивний тип Adaptive type

186 / 200
Для знеболювання використовують новокаїн, під дією якого нервове волокно втрачає здатність проводити збудження. Який мембранно-іонний механізм дії цього препарату? For pain relief, novocaine is used, under the influence of which the nerve fiber loses its ability to conduct excitation. What is the membrane-ion mechanism of action of this drug?

Блокування калієвих іоноселективних каналів Blocking of potassium ion-selective channels

Блокування натрієвих іоноселективних каналів Blocking of sodium ion-selective channels

Блокування кальцієвих іоноселектив-них каналів Blocking of calcium ion-selective channels

Блокування калій-натрієвого насоса Potassium sodium pump blocking

Блокування натрій-протонного насоса Na-proton pump blocking

187 / 200
У хворого 41-го року відзначається гіпонатріємія, гіперкаліємія, дегідратація, зниження артеріального тиску, м’язова слабкість, брадикардія, аритмія. З порушенням функцій яких гормонів це пов’язано? A 41-year-old patient has hyponatremia, hyperkalemia, dehydration, decreased blood pressure, muscle weakness, bradycardia, and arrhythmia. What hormones are this associated with 'linked?

Кортикостероїди Corticosteroids

Гормони мозкової речовини наднирників Adrenal medulla hormones

Гормони підшлункової залози Pancreatic hormones

Статеві гормони Sex hormones

Тиреоїдні Thyroid

188 / 200
При розтині хворої 28-ми років, що померла від уремії, виявлені збільшені строкаті нирки з осередками крововиливів. Патогістологічно в судинних клубочках виявлені гематоксилінові тільця, капілярні мембрани клубочків у вигляді дротяних петель, гіалінові тромби та осередки фібриноїдного некрозу. За патогенезом гіперчутливість якого типу лежить в основі описаної хвороби? At the autopsy of a 28-year-old patient who died of uremia, enlarged variegated kidneys with foci of hemorrhages were found. Pathohistologically, hematoxylin bodies were found in vascular glomeruli, capillary membranes of glomeruli in in the form of wire loops, hyaline thrombi and foci of fibrinoid necrosis. What type of hypersensitivity is the basis of the described disease in terms of pathogenesis?

Гіперчутливість III типу (імунокомплексна) Type III hypersensitivity (immunocomplex)

Гіперчутливість IV типу (клітинна цитотоксичність) Type IV hypersensitivity (cellular cytotoxicity)

Гіперчутливість I типу (анафілактична) Type I hypersensitivity (anaphylactic)

Гіперчутливість II типу (антитілозалежна) Type II hypersensitivity (antibody dependent)

Гіперчутливість V типу (гранулематоз) V type hypersensitivity (granulomatosis)

189 / 200
Хвора 37-ми років померла під час нападу експіраторної задухи, що був спричинений контактом з екзогенним алергеном (пилок амброзії). При гістологічному дослідженні в просвіті бронхів спостерігаються скупчення слизу, в стінці бронхів багато тучних клітин (лаброцитів), більшість з яких у стані де-грануляції, багато еозинофілів. До патогенезуякого типу реакцій гіперчутливості можна віднести описані зміни? A 37-year-old patient died during an attack of expiratory suffocation, which was caused by contact with an exogenous allergen (ambrosia pollen). During histological examination, accumulations of mucus are observed in the lumen of the bronchi , there are many mast cells (labrocytes) in the wall of the bronchi, most of which are in a state of degranulation, many eosinophils. The described changes can be attributed to the pathogenesis of what type of hypersensitivity reactions?

V типу (гранулематоз) Type V (granulomatosis)

I типу (анафілактична) Type I (anaphylactic)

III типу (імунокомплексна) Type III (immunocomplex)

II типу (антитілозалежна) Type II (antibody dependent)

IV типу (клітинна цитотоксичність) Type IV (cellular cytotoxicity)

190 / 200
Оперуючи на наднирниковій залозі, хірург зупиняє кровотечу з артерії, що відходить до наднирникової залози від черевної аорти. Назвіть цю артерію: When operating on the adrenal gland, the surgeon stops the bleeding from the artery leading to the adrenal gland from the abdominal aorta. Name this artery:

A. renalis A. renalis

A. phrenica inferior A. phrenica inferior

A. uprarenalis inferior A. uprarenalis inferior

A. suprarenalis superior A. suprarenalis superior

A. suprarenalis media A. suprarenalis media

191 / 200
Хворому на паратиф А інфекціоніст на 3-му тижні захворювання призначив повторне бактеріологічне дослідження. Який матеріал слід взяти для виділення збудника? On the 3rd week of the disease, the infectious disease specialist prescribed a repeat bacteriological examination for a patient with paratyphoid A. What material should be taken to isolate the pathogen?

Блювотні маси Vomiting masses

Ліквор Liquor

Харкотиння Snoring

Випорожнення Peel

Кров Blood

192 / 200
Пацієнтка тривалий час приймала снодійний засіб нітразепам. Після відміни препарату у неї розвинулись безсоння, зниження апетиту, агресивність. Як називається такий стан? The patient has been taking the hypnotic nitrazepam for a long time. After stopping the drug, she developed insomnia, decreased appetite, and aggressiveness. What is this condition called?

Тахіфілаксія Tachyphylaxis

Абстиненція Abstinence

Ейфорія Euphoria

Сенсибілізація Sensitization

Кумуляція Cumulative

193 / 200
Хворому на шизофренію призначено аміназин для купіювання психічного стану. Вкажіть механізм дії препарату: A patient with schizophrenia is prescribed aminazine to buy a mental state. Specify the mechanism of action of the drug:

- -

Блокада Д2-дофамінових рецепторів Blockade of D2-dopamine receptors

Стимуляція серотонінових рецепторів Serotonin receptor stimulation

Блокада ГАМК-рецепторів GABA receptor blockade

Стимуляція опіоїдних рецепторів Stimulation of opioid receptors

194 / 200
Хворому на пневмонію призначений антибіотик із групи макролідів. Вкажіть цей препарат: A patient with pneumonia is prescribed an antibiotic from the macrolide group. Specify this drug:

Тетрациклін Tetracycline

Азитроміцин Azithromycin

Ампіцилін Ampicillin

Гентаміцин Gentamicin

Стрептоміцин Streptomycin

195 / 200
В клініку доставлено чоловіка з травмою спини. Під час обстеження виявлено перелом хребців грудного відділу. Під час об’єктивного огляду нейрохірургом виявлено: нижче рівня перелому з правого боку відсутня глибока чутливість, з лівого боку – порушена температурна та тактильна чутливість. Яке ураження з боку спинного мозку є у хворого? A man was brought to the clinic with a back injury. During the examination, a fracture of the thoracic vertebrae was detected. During an objective examination by a neurosurgeon, it was found: below the level of the fracture on the right side, there is no deep sensitivity, on the left side - impaired temperature and tactile sensitivity. What damage does the patient have on the side of the spinal cord?

Анестезія Anesthesia

Парастезія Paraesthesia

Судомний синдром Convulsive syndrome

Хвороба Паркінсона Parkinson's disease

Синдром Броун Секара Brown Secard syndrome

196 / 200
Через 8 днів після хірургічної операції у пацієнта розвинувся правець. Лікар запідозрив, що причиною став контамінований збудником правця шовний матеріал, який був доставлений в бактеріологічну лабораторію. Яке живильне середовище необхідно використовувати для первинного посіву шовного матеріалу? After 8 days after the surgical operation, the patient developed tetanus. The doctor suspected that the cause was suture material contaminated with the tetanus pathogen, which was delivered to the bacteriological laboratory. What a nutrient medium must be used for primary sowing of suture material?

ЖСА ЖСА

Сабуро Saburo

Кітт-Тароцці Kitt-Tarozzi

Ендо Endo

Гіса His

197 / 200
Після проведеної операції на шиї хворий втратив чутливість в її передній ділянці. Яка гілка шийного сплетення була пошкоджена під час операції? After an operation on the neck, the patient lost sensitivity in its front area. Which branch of the cervical plexus was damaged during the operation?

Поперечний нерв шиї Transverse neck nerve

Надключичні нерви Supracral nerves

Малий потиличний нерв Small occipital nerve

Шийна петля Neck Loop

Великий вушний нерв Great auricular nerve

198 / 200
У пацієнта перед кардіологічною операцією зареєстровано тиск у всіх відділах серця. Який тиск в лівому шлуночку під час діастоли? The pressure in all parts of the heart was recorded in the patient before cardiac surgery. What is the pressure in the left ventricle during diastole?

100 мм рт.ст. 100 mm Hg

120 мм рт.ст. 120 mm Hg

40 мм рт.ст. 40 mm Hg

0 мм рт.ст. 0 mmHg

80 мм рт.ст. 80 mm Hg

199 / 200
При обстеженні жінки виявили перелом однієї з кісток черепа, ускладнений кровотечею з поперечної пазухи твердої оболонки мозку. Назвіть кістку, на якій розташовується борозна цієї пазухи: During the woman's examination, a fracture of one of the bones of the skull was discovered, complicated by bleeding from the transverse sinus of the dura mater. Name the bone on which the groove of this sinus is located:

Лобова Lobova

Скронева Temple

Клиноподібна Wedge

Тім’яна Timyana

Потилична Occipital

200 / 200
Батьки - глухонімі, але глухота у дружини залежить від аутосомно-рецесивного гена, а у чоловіка виникла внаслідок тривалого прийому антибіотиків у дитинстві. Яка імовірність народження глухої дитини в родині, якщо батько гомозиготний за аллелю нормального слуху? Parents are deaf and mute, but the wife's deafness depends on an autosomal recessive gene, and the husband's was due to prolonged use of antibiotics in childhood. What is the probability of a deaf child being born in the family , if the father is homozygous for the normal hearing allele?

10% 10%

0% 0%

25% 25%

100% 100%

75% 75%